Anda di halaman 1dari 70

CATATAN KULIAH

EKONOMI MINERAL
CATATAN KULIAH
EKONOMI MINERAL

YULI KUSUMAWATI, S.T., M.T.

Allah, tidak ada Tuhan selain Dia.


Yang Maha Hidup, yang terus-menerus mengurus (makhluk-Nya),
tidak mengantuk dan tidak tidur.
Milik-Nya apa yang ada di langit dan apa yang ada di bumi.
Tidak ada yang memberi syafaat di sisi-Nya tanpa izin-Nya.
Dia mengetahui apa yang ada di hadapan mereka dan apa yang ada di belakang mereka
dan mereka tidak mengetahui sesuatu apapun tentang ilmu-Nya melainkan apa yang Dia kehendaki.
Kursi-Nya meliputi langit dan bumi. Dan Dia tidak merasa berat memelihara keduanya,
dan Dia Maha Tinggi, Maha Besar.
(QS. Al Baqarah : 255)
DAFTAR ISI

1. DEFINISI DAN KONSEP DASAR 1


1.1. Ekonomi teknik dan pengambilan keputusaninvestasi 1
1.2. Karakteristik industri pertambangan 2
1.3. Konsep nilai waktu uang 3
1.4. Konsep kesetaraan 4
1.5. Aliran kas 5
2. KONSEP BIAYA 6
2.1. Pengertian biaya 6
2.2. Klasifikasi biaya 6
2.2.1. Biaya berdasarkan waktu 6
2.2.2. Biaya berdasarkan sifat penggunaannya 6
2.2.3. Biaya berdasarkan produk 7
2.2.4. Biaya berdasarkan volume produksi 8
2.3. Biaya dalam usaha pertambangan 9
2.3.1. Biaya kapital 9
2.3.2. Biaya operasi 10
3. KONSEP DAN RUMUSAN BUNGA 12
3.1. Bunga sederhana dan bunga majemuk 12
3.2. Suku bunga nominal dan suku bunga efektif 13
3.3. Minimum Attractive Rate of Return (MARR) 13
3.4. Waktu lipat dua 14
3.5. Rumus bunga majemuk 14
3.5.1. Single payment compound-amount 14
3.5.2. Single payment present-worth 15
3.5.3. Uniform series compound-amount 15
3.5.4. Sinking-fund deposit 15
3.5.5. Capital-recovery 16
3.5.6. Uniform series present-worth 16
3.5.7. Uniform gradient-series 16
4. INDIKATOR KELAYAKAN INVESTASI 18
4.1. Net Present Value (NPV) 18
4.2. Internal Rate of Return(ROR) 19
4.3. Benefit-Cost Ratio (BCR) 19
4.4. Present Value Ratio (PVR) 20
4.5. Payback Period (PBP) 20
5. ANALISIS MUTUALLY EXCLUSIVE 22
5.1. Analisis mutually exclusive dengan umur alternatif sama 22
5.2. Analisis mutually exclusive dengan umur alternatif berbeda 24
6. ANALISIS NON MUTUALLY EXCLUSIVE 27
6.1. Analisis non-mutually exclusive dengan umur alternatif sama 27
6.2. Analisis non-mutually exclusive dengan umur alternatif berbeda 29
7. DEPRESIASI, DEPLESI, AMORTISASI 32
7.1. Depresiasi 32
7.1.1. Straigth line depreciation (SLD) 33
7.1.2. Double declining balance depreciation (DDBD) 34
7.1.3. Sum of years digits depreciation (SOYD) 35
7.1.4. Sinking fund depreciation (SFD) 36
7.1.5. Unit of production depreciation (UPD) 38
7.2. Deplesi 39
7.2.1. Cost depletion 40
7.2.2. Percentage depletion 40
7.2.3. Pemilihan metode deplesi 41
7.3. Amortisasi 41
8. PENGARUH PAJAK DALAM ANALISIS LIFE CYCLE COSTING 43
8.1. Pengertian pajak 43
8.2. Cashflow setelah pajak 44
9. ANALISIS PENGGANTIAN 53
9.1. Konsep penggantian 53
9.2. Umur ekonomis 53
9.3. Membandingkan sewa atau beli 54
10. ANALISIS RESIKO DAN KETIDAKPASTIAN 59
10.1. Analisis sensitivitas 59
10.1.1. Analisis sensitivitas variabel tunggal 60
10.1.2. Analisis sensitivitas dengan pendekatan rentang 62
10.2. Analisis probabilitas 62
10.2.1. Teori probabilitas 62
10.2.2. Pohon keputusan 63

3
PENGANTAR

Alhamdulillahirabbilalamiin, berkat kuasa dan kasih sayang ALLAH SWT akhirnya Catatan Kuliah Ekonomi
Mineral ini bisa diselesaikan.

Catatan Kuliah Ekonomi Mineral ini diperuntukkan bagi mahasiswa teknik tingkat diploma maupun sarjana yang
ingin mendapatkan pemahaman mengenai prinsip-prinsip dasar dan metodologi ekonomi teknik dalam proses
pengambilan keputusan investasi pada kegiatan perencanaan dan pengembangan proyek-proyek keteknikan,
khususnya pada industri pertambangan dan pengelolaan sumberdaya mineral.

Materi dalam catatan kuliah ini merupakan rangkuman dari berbagai referensi yang disajikan secara ringkas
dan mudah dipahami namun tetap mencakup esensi dari setiap teori yang berkaitan dengan ekonomi teknik.
Disamping itu disajikan pula contoh soal sederhana untuk membantu pemahaman materi.

Mengingat keterbatasan yang ada, maka catatan kuliah ini masih banyak kekurangan dan perlu perbaikan. Oleh
karena itu masukan dan saran sangat diharapkan untuk penyempurnannya. Tak lupa penyusun menghaturkan
terima kasih kepada semua pihak yang telah membantu dalam menyelesaikan catatan kuliah ini. Selain itu
penghargaan yang besar penyusun berikan kepada suami dan anak-anak yang tidak surut atas memberikan
doa, semangat, dan dukungan.

Mudah-mudahan catatan kuliah ini menjadi amal kebaikan penyusun dan dapat memberikan manfaat yang
sebesar-besarnya.

Bandung, Juli 2014

Yuli Kusumawati S.T., M.T.

4
1. DEFINISI DAN KONSEP DASAR

Tujuan:

Memahami konsep ekonomi teknik dan aplikasinya dalam pengambilan keputusan investasi.
Memahami karakteristik industri pertambangan.
Memahami konsep nilai waktu uang dan kesetaraan.
Memahami konsep aliran kas.

1.1. Ekonomi teknik dan pengambilan keputusan investasi


Ekonomi teknik adalah ilmu yang berkaitan dengan teknik analisis kuantitatif yang berguna untuk memilih
satu alternatif yang lebih baik dari beberapa alternatif yang secara teknis layak.

Prosedur dalam analisis ekonomi teknik adalah:


1. Pengenalan masalah, perumusan, dan evaluasi.
2. Merancang alternatif yang layak.
3. Menyusun net cashflow dari masing-masing alternatif.
4. Pemilihan kriteria.
5. Menganalisis dan membandingkan semua alternatif.
6. Memilih alternatif yang tepat.
7. Memonitor kinerja dan mengevaluasi hasilnya.

Aplikasi prinsip-prinsip ekonomi teknik tidak hanya berguna dalam analisis kelayakan dari aspek finansial
terhadap proyek-proyek keteknikan tetapi juga membantu dalam pengambilan keputusan investasi pada
umumnya.

Investasi merupakan penukaran sejumlah dana dengan kemungkinan perolehan 100% (karena telah
dikuasai) dengan jumlah dana yang lebih besar tetapi kemungkinan perolehannya kurang dari 100%.
Investasi diperlukan untuk memulai suatu usaha atau mempertahankan dan meningkatkan kapasitas
produksi suatu usaha yang sedang berjalan.

Investasi memiliki dua faktor, yaitu:


1. Waktu
2. Resiko

Tujuan dilakukannya investasi adalah untuk memperoleh nilai lebih/keuntungan di masa depan dari modal
yang diinvestasikan. Modal yang dimaksud dapat berupa uang, barang modal, tanah, bangunan, teknologi,
ataupun sesuatu yang tidak riil, misalnya hak paten atau kemampuan manajerial. Dalam bidang
pertambangan, kapital umumnya berupa deposit bahan tambang dan modal.
Keputusan investasi modal akan mempunyai dampak jangka pendek dan jangka panjang bagi kelangsungan
perusahaan untuk dapat berkompetisi ataupun untuk tetap berproduksi. Keputusan investasi yang salah
tidak saja dapat mengurangi keuntungan perusahaan tetapi juga dapat menghentikan kegiatan perusahaan
sama sekali.

Keputusan investasi modal mempunyai dua bentuk tindakan utama, yaitu :


1. Mengalokasikan sejumlah modal untuk investasi proyek tertentu atau untuk menyediakan asset
produksi.
2. Memperoleh sejumlah modal untuk meningkatkan nilai perusahaan.

Ada lima langkah penting dalam pengambilan keputusan investasi, yaitu :


1. Mendefinisikan masalah

Yuli Kusumawati, Catatan Kuliah Ekonomi Mineral - 1


2. Menganalisis masalah
3. Mengembangkan alternatif solusi
4. Memutuskan solusi yang terbaik
5. Mengubah keputusan menjadi tindakan yang efektif

Analisis investasi diklasifikasikan menjadi tiga, yaitu :


1. Analisis ekonomi
Evaluasi terhadap kemakmuran relatif dari situasi-situasi investasi dari sudut pandang laba dan ongkos,
yaitu studi evaluasi terhadap keuntungan dari alternatif proyek.
2. Analisis finansial
Analisis finansial adalah evaluasi terhadap cara pendanaan terhadap investasi yang diusulkan.
Terdapat beberapa alternatif metode untuk pendanaan, di antaranya adalah dengan dana pribadi atau
perusahaan, meminjam dari bank, atau menawarkan saham pada publik.
Analisis finansial terbagi lagi menjadi tiga pengambilan keputusan penting yang harus dilakukan oleh
sebuah perusahaan, yaitu :
a. Keputusan deviden
b. Keputusan keuangan
c. Keputusan investasi
Ketiga keputusan ini saling terkait satu sama lain dan mempengaruhi analisis investasi secara
keseluruhan.
3. Analisis intangible
Evaluasi terhadap faktor-faktor yang mempengaruhi investasi tetapi sukar diukur secara kuantitatif.
Contoh faktor intangible antara lain perijinan, pertimbangan keamanan, opini publik, pertimbangan
politik, faktor ekologi dan lingkungan, dan ketidakpastian kondisi peraturan pajak.

Ketiga jenis analisis investasi ini mutlak harus dilakukan, karena sering terjadi suatu alternatif yang hasil
analisis ekonominya baik, ternyata tidak cukup baik jika dilihat dari analisis finansial dan intangible-nya.
Sebagai contoh, suatu proyek ditolak dari analisis finansial bila dana internalnya tidak mencukupi untuk
membiayai proyek dan juga tidak dapat memperoleh pinjaman dana dari luar dengan tingkat bunga yang
sesuai. Faktor intangible yang dapat mengakibatkan suatu proyek ditolak secara analisis ekonomi pada
umumnya adalah karena opini publik dan masalah lingkungan (polusi udara, tanah, dan air). Karenanya
pengaruh analisis finansial dan intangible terhadap analisis ekonomi harus diperhatikan secara seksama
karena analisis-analisis tersebut mempunyai pengaruh yang sangat besar terhadap pemilihan alternatif
investasi.

1.2. Karakteristik industri pertambangan


Industri pertambangan merupakan bagian dari dunia industri umum namun industri pertambangan
mempunyai karakteristik khusus yang berbeda dengan industri lainnya. Pemahaman tentang karakteristik
khusus ini penting untuk melakukan analisis kelayakan suatu proyek/investasi tambang.

Karakteristik Industri pertambangan antara lain:


1. Modal Besar
Besarnya modal yang dibutuhkan untuk industri tambang bervariasi, tergantung dari jenis bahan
tambang, metode penambangan, skala penambangan, lokasi dan parameter lainnya.
2. Periode Pra Produksi yang Panjang
Lama periode pra produksi tergantung dari metode penambangan, metode pengolahan, ukuran dan
letak deposit, kompleksitas operasi, dan kendala lingkungan. Periode pra produksi ini berkisar antara 3
12 tahun. Periode pra produksi yang panjang akan berdampak terhadap besar modal yang dibutuhkan
dan terhadap tingkat pengembalian modal.
3. Beresiko Tinggi

Yuli Kusumawati, Catatan Kuliah Ekonomi Mineral - 2


Disamping resiko yang berhubungan dengan kebutuhan modal yang besar serta masa pra produksi
yang lama, terdapat resiko lain yang mempengaruhi keputusan investasi pada industri tambang, yaitu:
resiko geologi, resiko engineering dan konstruksi, reiko ekonomi, resiko politik, dan resiko pasar mineral.
4. Sumber daya alam yang tak dapat diperbaharui (nonrenewable resources)
Implikasi dari sumber daya alam yang tidak dapat diperbaharui ini adalahterhadap pendapatan utama
perusahaan yang diperoleh dari penjualan bahan tambang, yang mengakibatkan umur tambang
tergantung dari jumlah cadangan dan tingkat produksi sehingga dibutuhkan eksplorasi kontinyu untuk
menemukan deposit baru.
5. Mendorong pertumbuhan ekonomi
Dikarenakan letak aktivitas penambangan banyak terdapat di daerah terpencil, hal ini akan dapat
memberikan dampak positif terhadap aktivitas ekonomi masyarakat setempat sehingga dapat
mendorong pertumbuhan ekonomi di daerah tersebut.
6. Dampak terhadap lingkungan
Kegiatan eksploitasi bahan tambang akan mengubah bentang alam sehingga berdampak buruk
terhadap keadaan lingkungan. Oleh karena itu tingkat kepedulian industri tambang terhadap
lingkungan harus tinggi. Reklamasi merupakan salah satu upaya untuk mengurangi dampak lingkungan
dari kegiatan penambangan dan pengolahan.
7. Sifat indestructibility of product
Konsekuensi dari sifat ini adalah munculnya pasar sekunder dan dapat mengurangi prosentase
kebutuhan akan bijih/bahan tambang. Daur ulang logam sering dipertimbangkan lebih menguntungkan
dibandingkan menambang bijih untuk dijadikan logam.

1.3. Konsep nilai waktu uang


Konsep nilai waktu uang (time value of money) mengacu pada biaya kesempatan (opportunity cost) dalam
menghasilkan pendapatan pada saat sekarang. Bila sejumlah dana tidak digunakan untuk membiayai suatu
proyek, maka dana tersebut akan dapat digunakan untuk membiayai proyek lain. Dengan kata lain jika suatu
dana ditanamkan seluruhnya pada suatu proyek maka akan menghilangkan kesempatan untuk
mendapatkan penghasilan (return) dari proyek lain yang tidak dipilih.

Nilai sesuatu (termasuk uang) berubah seiring dengan berjalannya waktu. Perjalanan waktu pada umumnya
mengurangi nilai kecuali pada benda antik dan benda yang mempunyai umur layanan (service life) tak
terbatas, seperti tanah.

Beberapa hal yang menyebabkan nilai uang dipengaruhi oleh waktu adalah:
1. Uang memiliki purchasing power (daya beli).
Secara umum uang digunakan untuk melakukan transaksi, sehingga semua komoditi harus dinilai
berdasarkan nilai tukarnya terhadap uang.
Daya beli uang menurun disebabkan oleh:
Inflasi
Perubahan pola supply (pasokan) dan demand (permintaan)
Perubahan struktur ekonomi
Sebagai contoh harga premium sekarang Rp 6.500/liter, artinya 1 liter premium nilainya Rp
6.500.Beberapa tahun yang lalu harga premium masih Rp 4.500/liter, atau dikatakan nilai uang
mengalami penurunan terhadap premium.
2. Uang memiliki earning power (kemampuan menghasilkan).
Sejumlah uang yang disimpan di bank akan bertambah dengan adanya bunga. Sebagaimana meminjam
barang, bunga (interest) dianggap sebagai sewa atas pemakaian uang tersebut.

1.4. Konsep kesetaraan


Konsep kesetaraan (equivalence) menyatakan bahwa sejumlah uang pada satu waktu tertentu setara
nilainya dengan uang dalam jumlah yang berbeda pada waktu yang berbeda apabila diberikan discount rate
tertentu. Konsep kesetaraan dapat digunakan untuk mengevaluasi rencana investasi. Untuk keperluan

Yuli Kusumawati, Catatan Kuliah Ekonomi Mineral - 3


tersebut maka seluruh pendapatan dan pengeluaran proyek di masa yang akan datang dihitung nilainya
pada saat sekarang (present worth).

Sebagai contoh uang sejumlah Rp 10 juta yang disimpan di bank dengan bunga 20%/tahun dapat diambil
dalam beberapa cara, antara lain:
a. Disimpan hari ini dan baru diambil tahun depan dengan jumlah Rp 12 juta, dengan rincian pokok
simpanan Rp 10 juta ditambah bunga Rp 2 juta.
b. Diambil bunganya saja Rp 2 juta tiap tahun selama lima tahun, dan pada akhir tahun kelima pokok
simpanannya diambil. Jumlah uang yang diperoleh selama lima tahun tersebut adalah Rp 20 juta,
dengan rincian bunga Rp 2 juta x 5 = Rp 10 juta dan pokok simpanan Rp 10 juta.
c. Diambil dalam jumlah yang sama setiap tahunnya selama sepuluh tahun sebesar Rp 2,385 juta. Jumlah
uang yang diperoleh selama sepuluh tahun tersebut adalah Rp 23,85 juta, yaitu Rp 2,385 juta x 10.
Ketiga jumlah nominal uang tersebut yaitu Rp 12 juta, Rp 20 juta, dan Rp 23,85 juta pada dasarnya setara
dengan Rp 10 juta pada saat sekarang. Kesetaraan tersebut terjadi dengan penggunaan bunga 20% per
tahun.

1.5. Aliran kas


Aliran kas (cashflow) adalah aliran uang yang terdiri atas cash-in (uang masuk) dan cash-out (uang keluar)
yang terjadi selama periode operasi tertentu, misalnya bulan atau tahun. Cashflow digunakan untuk
menggambarkan pergerakan uang suatu perusahaan pada satuan periode tertentu.

Aliran kas terdiri dari:


Uang masuk (cash in), umumnya berasal dari penjualan produk atau manfaat (benefit) yang terukur.
Uang keluar (cash out), merupakan kumulatif dari biaya-biaya (cost) yang dikeluarkan.
Net cashflow adalah jumlah uang masuk dikurangi jumlah uang keluar. Net cashflow yang negatif artinya
defisit, yaitu cash-out lebih besar daripada cash-in.

Discounted cashflow adalah nilai cashflow yang dibawa ke nilai sekarang (present worth) sesuai dengan
discount rate yang diberikan. Istilah discount mengacu pada pengurangan nilai, yaitu jika sejumlah tertentu
uang dimasa datang dihitung nilainya sekarang maka nilai uang tersebut akan lebih kecil (berkurang).

Cashflow dapat digambarkan dalam bentuk:


1. Tabel yang terdiri paling sedikit tiga kolom, yaitu kolom waktu, kolom pengeluaran, dan kolom
pendapatan.
2. Diagram yang ditunjukkan dengan anak panah yang mengarah ke atas untuk cash-in dan anak panah
yang mengarah ke bawah untuk menggambarkan cash-out.

Contoh 1-1:
Pada tahun awal (tahun ke-0) biaya yang dikeluarkan adalah Rp 10 juta. Tahun ke-1 dikeluarkan lagi biaya Rp
2 juta selain diterima pendapatan (revenue) Rp 3 juta. Pada tahun ke-2, 3, 4, dan 5 diperoleh pendapatan
masing-masing Rp 5 juta. Gambarkan undiscounted cashflow perusahaan tersebut dalam bentuk tabel dan
diagram.
Jawab:
Tabel undiscounted cashflow:

Tahun Pendapatan(cash-inflow) Pengeluaran (cash-outflow) Net cashflow CummulativeCashflow


0 0 10.000.000 -10.000.000 -10.000.000
1 3.000.000 2.000.000 1.000.000 -9.000.000
2 5.000.000 0 5.000.000 -4.000.000
3 5.000.000 0 5.000.000 1.000.000
4 5.000.000 0 5.000.000 6.000.000
5 5.000.000 0 5.000.000 11.000.000

Yuli Kusumawati, Catatan Kuliah Ekonomi Mineral - 4


Jumlah 23.000.000 12.000.000 11.000.000

Diagram cashflow:

5.000.000 3.000.000 5.000.000


5.000.000 5.000.000

2 0 3

2.000.000 10.000.0000

Dari tabel di atas terlihat pada akhir tahun ke-5, proyek tersebut menghasilkan keuntungan bersih sebesar
Rp 11.000.000.

Secara umum cash flow dikelompokkan menjadi:


1. Cashflow uniform, adalah cashflow dimana besar cash-in dan cash-out tiap periode adalah seragam
(tetap).
2. Cashflow gradient uniform, adalah cashflow dimana besar cash-in atau cash-out bertambah atau
berkurang dengan gradien tetap.
3. Cashflow seri geometrik, adalah cashflow dimana besar cash-in atau cash-out bertambah atau
berkurang dengan presentase tetap.

Yuli Kusumawati, Catatan Kuliah Ekonomi Mineral - 5


2. KONSEP BIAYA
Tujuan:
Memahami konsep biaya dan klasifikasinya secara umum.
Mampu menghitung biaya produksi dengan metode yang sesuai.
Mampu menganalisis biaya produksi sehingga bisa memperbaiki efisiensinya.
Mengetahui komponen biaya yang ada dalam usaha pertambangan.

2.1. Pengertian biaya


Biaya adalah jumlah uang yang harus dikeluarkan untuk memproduksi sesuatu (cost of production) atau
harga yang harus dikeluarkan untuk mendapatkan sesuatu (supply price).
Konsep biaya dalam analisis ekonomi teknik perlu dipahami karena semua analisis yang dilakukan
didasarkan atas perkiraan arus kas yang dikembangkan berdasarkan perkiraan biaya (cost estimate) dan
proyeksi pendapatan (revenue projection).

2.2. Klasifikasi biaya


Biaya dapat kelompokkan berdasarkan klasifikasi berikut (Giatman, Ekonomi Teknik):
a. Berdasarkan waktunya, dibedakan menjadi:
Biaya masa lalu
Biaya perkiraan
Biaya sebenarnya
b. Berdasarkan sifat penggunaannya, dibedakan menjadi:
Biaya investasi
Biaya operasi
Biaya perawatan
c. Berdasarkan produknya, dibedakan menjadi:
Biaya produksi/pabrikasi
Biaya komersial
d. Berdasarkan volume produk, dibedakan menjadi:
Biaya tetap
Biaya tidak tetap
Biaya semi variable

2.2.1. Biaya berdasarkan waktu


Biaya berdasarkan waktu dibedakan menjadi:
1) Biaya masa lalu (hystorical cost), yaitu biaya yang secara riil telah dikeluarkan yang dibuktikan dengan
catatan historis pengeluaran kegiatan.
Biaya historis digunakan sebagai dasar untuk:
Penyusunan/perkiraan biaya masa datang.
Pertanggungjawaban atau audit terhadap biaya-biaya yang telah dikeluarkan.
2) Biaya perkiraan (predictive cost), yaitu perkiraan biaya yang akan dikeluarkan bila suatu kegiatan
dilaksanakan.
Biaya perkiraan digunakan untuk:
Memperkirakan pemakaian biaya dalam merealisasikan suatu kegiatan.
Menganalisis efektivitas dan efisiensi biaya terhadap kegiatan yang akan dilaksanakan.
3) Biaya aktual (actual cost), yaitu biaya yang sebenarnya dikeluarkan. Biaya ini perlu diperhitungkan jika
rentang waktu antara pembelian dengan proses produksi atau penjualan cukup lama sehingga
terpengaruh dengan perubahan harga pasar.
Biaya aktual dibedakan menjadi:
Biaya riil (expense), yaitu biaya yang dikeluarkan secara riil.
Biaya semu (sunk cost), yaitu biaya yang ditanggung tetapi tidak pernah dikeluarkan secara riil.
Misalnya, selisih harga pembukuan aset yang akan dilikuidasi dengan harga pasar.
Biaya kesempatan (opportunity cost), yaitu biaya yang ditanggung akibat kelalaian dalam
memanfaatkan peluang untuk mendapatkan keuntungan yang lebih besar.
Metode perhitungan biaya aktual antara lain:
First in first out (FIFO)
Last in first out (LIFO)
Metode rata-rata (average method)
Metode harga standar (standard price method)

Yuli Kusumawati, Catatan Kuliah Ekonomi Mineral - 6


2.2.2. Biaya berdasarkan sifat penggunaannya
Biaya berdasarkan sifat penggunaannya dikelompokkan menjadi tiga, yaitu:
1) Biaya investasi (investment cost), yaitu biaya yang ditanamkan untuk membangun dan
mengembangkan suatu usaha sehingga siap beroperasi. Biaya ini bisanya dikeluarkan pada awal
kegiatan dengan jumlah yang relatif besar dan mempunyai dampak jangka panjang untuk
keberlangsungan usaha tersebut.
Contoh:
Pembelian tanah atau hak penggunaan lahan.
Pembangunan pabrik, gedung, dan infrastuktur pendukung.
Pengadaan mesin dan peralatan produksi.
Pengadaan alat transportasi
Pengadaan perabotan kantor dan sarana pendukung administrasi.
Pendidikan dan pelatihan sumberdaya manusia.
2) Biaya operasi (operational cost), yaitu biaya yang dikeluarkan untuk menjalankan kegiatan usaha. Biaya
ini biasanya dikeluarkan secara rutin atau berkala dalam jumlah yang relatif sama sesuai dengan jadwal
kegiatan/produksi.
Biaya operasi digunakan antara lain untuk:
Pembelian bahan baku
Pembelian bahan pendukung
Pembayaran upahkaryawan
Pengeluaran untuk keperluan organisasi dan administrasi.
3) Biaya perawatan (maintenance cost), yaitu biaya yang digunakan untuk menjamin kinerja fasilitas atau
peralatan agar selalu prima dan siap untuk digunakan.
Sifat pengeluaran ini umumnya dibedakan menjadi dua, yaitu:
a. Biaya perawatan rutin (preventive maintenance)
b. Biaya perawatan mendadak (curative maintenance)

2.2.3. Biaya berdasarkan produk


Biaya berdasarkan produk dikelompokkan menjadi dua, yaitu:
1) Biaya produksi (production cost) atau biaya pabrikasi (factory cost), yaitu biaya yang langsung
berkaitan dengan proses produksi.
Biaya produksi terdiri dari komponen berikut:
a. Biaya utama (prime cost), yaitu biaya untuk bahan langsung dan tenaga kerja langsung yang
berhubungan dengan proses produksi.
b. Biaya overhead, yaitu biaya untuk bahan tak langsung, tenaga kerja tak langsung, dan biaya tak
langsung lainnya.
c. Biaya konversi (conversion cost), yaitu biaya pengubahan bahan bahan baku menjadi barang jadi.
Biaya bn biaya overhead dapat digabung ke dalam kelompok biaya konversi.
2) Biaya komersial (commercial cost)
Biaya komersial adalah biaya di luar biaya produksi yang digunakan untuk keperluan penjualan produk.
Biaya komersial meliputi:
a. Biaya umum dan administrasi (general and administration cost), contoh gaji karyawan dan
pimpinan, biaya untuk keperluan administrasi, dan telekomunikasi.
b. Biaya pemasaran (marketing cost), meliputi biaya distribusi, dan promosi.
c. Pajak usaha dan perusahaan (company tax), meliputi semua pajak dan retribusi yang dikeluarkan
perusahaan.
Tujuan perhitungan biaya berdasarkan produk antara lain:
Memproyeksikan biaya produksi dan harga jual produk.
Mengetahui komposisi komponen biaya produksi dan biaya keseluruhan produk.
Untuk menganalisis struktur biaya produk yang ideal guna memperbaiki struktur pembiayaan melalui
konsep pusat-pusat pembiayaan.

2.2.4. Biaya berdasarkan volume produksi


Biaya berdasarkan volume produksi dibedakan menjadi:
1) Biaya tetap (fixed cost), yaitu biaya yang dikeluarkan dengan jumlah yang relatif sama walaupun
volume produksi berubah dalam batas tertentu. Contoh:gaji karyawan tetap, biaya penyusutan, pajak
bumi dan bangunan, biaya sewa, biaya listrik untuk penerangan, telepon, dan air.

Yuli Kusumawati, Catatan Kuliah Ekonomi Mineral - 7


2) Biaya tidak tetap (variable cost), yaitu biaya yang besarnya berubah secara proporsional dengan jumlah
produk yang dibuat. Contoh: biaya bahan baku, tenaga kerja langsung yang digaji berdasarkan volume
kerja, biaya penyusutan yang tergantung dengan volume produksi.
3) Biaya semi variable (semi variable cost), yaitu biaya yang berubah tidak proporsional terhadap volume
produksi. Contoh: perubahan volume produksi yang melampaui kapasitas mesin, sehingga diperlukan
biaya penambahan kapasitas mesin, atau biaya perbaikan mesin.

Contoh 2-1:
PT Insan Kreatif mendapat pesanan bangku untuk suatu sekolah. Di pasaran harga bangku (S) sesuai dengan
spesifikasi teknis yang diajukan sekolah itu adalah Rp 450.000/unit. Jika bangku itu dibuat sendiri maka
diperlukan biaya-biaya sebagai berikut:
Harga kayu = Rp 950.000/m3
Paku = Rp 20.000/kg
Bahan finishing = Rp 100.000/lt
Upah tukang = Rp 80.000/orang/hari
Upah pembantu tukang = Rp 50.000/orang/hari
Satu unit bangku membutuhkan 0,20 m3 kayu, 0,01kg paku, bahan finishing 0,1lt, dan diselesaikan selama
0,5 hari kerja oleh tukang ditambah1 hari kerja oleh pembantunya.Selain itu perusahaan memerlukan
bengkel kerja dengan biaya sekitar Rp 1,5 juta dan pengadaan peralatan kerja Rp 2,2 juta.

Kelompokkan biaya berdasarkan biaya tetap dan biaya variable. Hitunglah berapa jumlah minimal pesanan
bangku yang harus diterima perusahaan untuk mencapai titik impas (break even point/BEP)?
Jawab:
Biaya tetap:
Membangun bengkel kerja = Rp 1.500.000
Pengadaan alat kerja = Rp 2.200.000
Jumlah fixed cost (FC) = Rp 3.700.000
Biaya variable per unit bangku:
Bahan kayu = 0,2 m3 x Rp 950.000/m3 = Rp 190.000
Paku = 0,01kg x Rp 20.000/kg = Rp200
Bahan finishing = 0,1 lt x Rp 100.000/lt = Rp 10.000
Upah tukang = 0,5 hari x Rp 80.000/orang/hari = Rp 40.000
Upah pembantu tukang = 1 hari x Rp 50.000/orang/hari = Rp 50.000
Jumlah variable cost (VC) = Rp 290.200
Jumlah produksi minimal untuk mencapai titik impas (BEP) adalah:

FC
BEP=
SVC

Rp 3.700 .000
BEP= =23,15 24 unit
Rp 450.000Rp 290.200

2.3. Biaya dalam usaha pertambangan


Secara umum biaya dalam usaha pertambangan dibedakan menjadi dua, yaitu:
1. Biaya kapital (biaya investasi), terdiri dari dua komponen yaitu:
1) Biaya kapital tetap
2) Modal kerja
2. Biaya operasi, terdiri dari tiga komonen yaitu:
1) Biaya operasi langsung

Yuli Kusumawati, Catatan Kuliah Ekonomi Mineral - 8


2) Biaya operasi tak langsung
3) Biaya overhead

2.3.1. Biaya kapital


Biaya kapital (capital cost) adalah jumlah biaya yang dibutuhkan untuk membuat suatu endapan bahan
galian yang berada di dalam bumi menjadi produk tambang yang dapat dijual.
Biaya kapital terdiri dari dua komponen penting, yaitu:
1) Biaya kapital tetap (initial investment)
Jika tambang yang akandikerjakan merupakan tambang baru, maka biaya tetap biasanya terdiri dari
komponen-komponen berikut:
Land Acquisition (pembebasan lahan), biayanya tergantung kepada luas dan lokasi lahan.
Konstruksi pra-penambangan (pengupasan tanah penutup, dan sebagainya).
Pembangunan tambang/masa konstruksi
Analisa Dampak Lingkungan.
Peralatan tambang, bangunan, sarana lain.
Peralatan pabrik, bangunan, sarana lain.
Sarana penunjang (jalan, listrik, perumahan, sarana olahraga, instalasi air, dan sebagainya).
Jasa perancangan dan konsultasi.
Contingency.
2) Modal kerja (working capital).
Modal kerja adalah biaya yang digunakan untuk memulai produksi sebelum perusahaan mendapatkan
uang dan hasil penjualan produknya. Besarnya modal kerja umumnya adalah 25% dari biaya operasi
atau mencukupi kebutuhan operasi selama 3-6 bulan.
Modal kerja umumnya terdiri dari komponen-komponen biaya sebagai berikut:
a. Persediaan
Bahan baku, berupa dimana cadangan endapan mineral/bijih yang ekonomis yang belum
dilaksanakan proses penambangan.
Suku cadang, yang berguna apabila terjadi ker usakan pada alat-alat penambangan.
Supplies, merupakan stock dari suatu perusahan tambang yang berupa perlengkapan habis sekali
pakai seperti perlengkapankantor, bahan bakar, bahan pelumas, dll.
Bahan dalam proses (materialsin process), berupa endapan mineral/bijih yang sedang atau dalam
proses penambangan atau dalam proses pengolahan (mineral dressing).
Bahan jadi/produk tambang, merupakan bahan galian/bijih yang telah melalui proses pengolahan
yang siap dijual.
b. Piutang dagang
Piutang dagang merupakan suatu modal kerja yang dapat ditarik sewaktu-waktu dari pihak
kedua sesuai dengan perjanjian dagang (seperti pembayaran diakhir transaksi penjualan komoditas
dagang).
c. Hutang dagang
Merupakan modal kerja yang diperoleh dari or ang lain dalam bentuk pinjaman yang bernilai
ekonomis, yang harus dibayar oleh kita apabila telah jatuh tempo sesuai dengan perjanjian dagang
kedua belah pihak
d. Kas, dan lain-lain.
Merupakan cadangan uang yang disimpan yang berguna untuk membiayai kehidupan tambang
sehari-hari, dengan periode waktu yang relatif singkat.

2.3.2. Biaya Operasi


Biaya operasi(operating cost) adalah segala macam biaya yang harus dikeluarkanagar proyek penambangan
dapat beroperasi/berjalan dengan normal. Dalam suatu operasi penambangan, keseluruhan biaya
penambangan terdiri dari banyak komponenbiaya yang merupakan akibat dari masing-masing tahap
kegiatan. Besar kecilnya biaya penambanganakantergantung pada perancangan teknis sistem
penambangan, jenis dan jumlah alat yang digunakan.

Yuli Kusumawati, Catatan Kuliah Ekonomi Mineral - 9


Aspek teknis dan aspek ekonomis tidak dapat berjalan sendiri-sendiri, keduanyaakanselalu saling
mempengaruhi. Perkiraan biaya investasi alatakan tergantung pada jumlah alat yang dipergunakan dan
kapasitas alat yang dipilih. Demikian pula biaya produksi merupakan fungsi dari kapasitas alat yang dipakai.
Biaya penambangan yang rendah dapat dicapai jika rancangan teknis dapat dioptimalkan dengan
memperhatikan pemilihan dan jumlah alat yang akan digunakan, yaitu yang dapat memberikan biaya
produksi per ton yang rendah.

Secara umum biaya operasi dibagi menjadi tiga komponen biaya, yaitu:
1) Biaya operasi langsung
Biayaoperasi langsung merupakan biaya utama dan berkaitan langsungdenganproduk yang dihasilkan.
Walaupun komponen biaya operasi langsung dari satu tambang ke tambang yang lain bervariasi, akan
tetapi pada umumnya terdiri dari:
Upah pekerja (pekerja lapangan, pengawas lapangan, dan sebagainya)
Bahan bakar (bahan bakar, oli, dan sebagainya)
Royalti.
Persiapan daerah produksi/permukaan kerja
2) Biaya operasi tak langsung
Biaya operasi tak langsung adalah pengeluaran-pengeluaran yang tak terpengaruholeh produksi yang
dihasilkan. Biaya operasi langsung terdiri dari:
Gaji pekerja (administrasi, keamanan, teknisi, jurubayar, petugas kantor, bengkel dan sebagainya).
Asuransi.
Penyusutan alat.
Pajak.
Reklamasi daerah bekas tambang.
Perjalanan bisnis, rapat, sumbangan-sumbangan.
Keperluan kantor.
Humas dan sebagainya.
3) Biaya overhead
Biaya overhead dapat/tidak dapat dimasukkan sebagai ko mponen biaya operasi tetapi biaya-biaya ini
berpengaruh terhadap total biaya produksi walaupun umumnya mencerminkan biaya-biaya diluar
tambang/biaya-biaya perusahaan. Overhead biasanya dikelompokkan menjadi:
a. Penjualan.
b. Administrasi kantor pusat.

Yuli Kusumawati, Catatan Kuliah Ekonomi Mineral - 10


3. KONSEP DAN RUMUSAN BUNGA
Tujuan:
Memahami konsep bunga sederhana dan bunga majemuk.
Mampu menghitung suku bunga nominal, periode, dan efektif.
Mampu menerapkan rumus bunga majemuk yang sesuai untuk menghitung kesetaraan nilai uang pada
berbagai titik waktu.

3.1. Bunga sederhana dan bunga majemuk


Bunga (interest) adalah uang yang dibayarkkan atas penggunaan sejumlah pinjaman. Bunga juga bisa
berarti uang yang diperoleh dari investasi sejumlah modal tertentu sebagai bentuk kompensasi resiko yang
timbul dari investasi tersebut.
Tingkat/suku bunga (interest rate) adalah rasio antara bunga yang dibayarkan terhadap pokok dalam suatu
periode waktu.
Periode bunga (interest period) adalah interval waktu yang dijadikan dasar dalam perhitungan bunga.

Secara umum perhitungan bunga dikelompokkan menjadi dua, yaitu:


1. Bunga sederhana (simple interest), yaitu bunga yang dihitung dari pokok uang/pinjaman selama periode
pinjaman.
I=Pin
Dimana:
I = bunga (interest)
P = pokok (principal)
i = tingkat/suku bunga (interest rate)
n = periode pinjaman
Sehingga jumlah uang/pinjaman pada akhir periode (F) adalah:
F = Pokok + Bunga
=P+Pin
= P (1 + i n)

Contoh 3-1:
Seorang petani meminjam uang di KUD sebesar Rp 10.000.000 dengan bunga 10%/tahun dengan
perhitungan bunga sederhana. Berapa hutang yang harus dibayar pada akhir tahun keempat?
Jawab:
F = Rp 10.000.000 x (1 +(10% x 4))
= Rp 14.000.000

2. Bunga majemuk (compound interest), yaitu bunga yang dihasilkan dari pokok pinjaman dibungakan lagi
bersama-sama dengan pokok pinjaman.
I = P(1 + i)n-1 . i
Sehingga jumlah uang/pinjaman pada akhir periode (F) adalah:
F = Pokok + Bunga
= P + P(1 + i)n-1 . i
= P (1 + i)n

Contoh 3-2:

Yuli Kusumawati, Catatan Kuliah Ekonomi Mineral - 11


Seorang karyawan meminjam uang di Bank sebesar Rp 10.000.000 dengan bunga 10%/tahun dengan
perhitungan bunga majemuk. Berapa hutang yang harus dibayar pada akhir tahun keempat?
Jawab:
F = Rp 10.000.000 (1 + 10%)4
= Rp 14.641.000

Dari kedua contoh di atas ternyata bunga majemuk nilainya lebih tinggi dari bunga sederhana. Dalam
pembahasan pemilihan alternatif atau evaluasi rencana investasi digunakan bunga majemuk.

3.2. Suku bunga nominal dan suku bunga efektif


Suku bunga nominal (nominal rate) per tahun (r) adalah perkalian suku bunga per periode (i) dengan jumlah
pemajemukan dalam setahun (m).
r = i. m
Suku bunga efektif (effective rate) disimbolkan E adalah suku bunga sesungguhnya dalam satu tahun yang
tepat dibayarkan terhadap sejumlah uang yang disimpan atau dipinjam.

E = (1 + i)m -1 atau E = (1+ (r/m))m 1 atau E = (F/P)1/n 1

Contoh 3-3:
Bank ABC membebankan suku bunga sebesar 1,25 % per bulan pada semua saldo rekening yang belum
terbayar. Berapakah suku bunga nominal dan suku bunga efekti per tahunnya?
Jawab:
Suku bunga periode (i) = 1,25% per bulan
Jumlah pemajemukan/th (m) = 12
Suku bunga nominal (r) = 1,25% x 12 = 15% per tahun
Suku bunga efektif (E) = (1 + (0,15/12))12 1 = 16,18% per tahun

Tabel 3.1. Tingkat suku bunga efektif per tahun dari berbagai suku bunga nominal.

3.3. Minimum Attractive Rate of Return (MARR)


MARR adalah laju pengembalian minimum yang dikehendaki oleh investor.MARR tergantung pada
lingkungan, jenis kegiatan, tujuan dan kebijakan organisasi, dan tingkat resiko masing-masing proyek.

Pertimbangan dalam menentukan MARR antara lain:


a. Jika perusahaan beroperasi dengan modal pinjaman, maka besarnya MARR harus lebih besar daripada
bunga yang dibayarkan pada pinjaman itu.
b. Jika modal datang dari berbagai sumber (dana-dana yang berasal dari perusahaan sendiri, hutang
jangka panjang maupun jangka pendek, dan sumber-sumber keuangan lainnya), maka penentuan biaya
modal rata-rata bisa dipakai sebagai basis untuk menentukan besarnya MARR.

Yuli Kusumawati, Catatan Kuliah Ekonomi Mineral - 12


c. Besarnya MARR harus setara dengan kecepatan pertumbuhan dari kekayaan perusahaan yang telah
ditetapkan oleh pimpinan perusahaan.
d. MARR untuk proyek yang resikonya tinggi harus lebih besar daripada proyek yang resikonya rendah.
e. Perusahaan yang lebih bonafide (lebih banyak kesempatan mendapatkan proyek) biasanya menetapkan
MARR lebih tinggi daripada perusahaan sejenis yang belum punya nama.

3.4. Waktu lipat dua


Waktu lipat dua (doubling time) adalah waktu yang diperlukan agar uang yang ditanamkan hari ini menjadi
dua kali lipat. Untuk memperkirakan waktu ini dengan cepat digunakan angka ajaib (magic number) 72.

Waktu lipat dua = 72 / (suku bunga majemuk per periode x 100)

Akan tetapi penggunaan magic number ini akan salah jika periode pemajemukan kurangdari tiga (n3) atau
suku bunga (i) di atas 30%

Contoh 3-4:
Berapa tahun uang yang disimpan di bank menjadi dua kali lipat jika suku bunga yang berlaku adalah 8%
per tahun?
Jawab:
n = 72/(8% x 100) = 9 tahun

3.5. Rumus bunga majemuk


Notasi dalam rumus bunga:
i = suku bunga tiap periode bunga
n = jumlah periode bunga
P = nilai uang sekarang
F = nilai uang mendatang
A = pembayaran seri tahunan yang seragam
G = pembayaran seri gradient

3.5.1. Single payment compound-amount


Digunakan untuk menentukan nilai mendatang (F) dari sejumlah uang pada saat sekarang (P) selama
periode (n) tertentu dan pada tingkat suku bunga (i) tertentu.

Contoh 3-5:
Seorang ayah mendepositokan uang Rp 100 juta sekarang di bank dengan tingkat bunga 12%/ tahun. Uang
tersebut akan diambil pada akhir tahun kelima, saat anaknya akan masuk universitas. Berapa jumlah uang
yang akan diperoleh sang ayah?
Jawab:
F = Rp 100 juta (1 + 0,12)5
= Rp 100 juta (F/P )
12%, 5

= Rp 100 juta (1,7623)= Rp 176.230.000

3.5.2. Single payment present-worth

Yuli Kusumawati, Catatan Kuliah Ekonomi Mineral - 13


Digunakan untuk menentukan nilai sekarang dari sejumlah uang yang diterima pada waktu mendatang
selama periode tertentu dan tingkat bunga tertentu.

Contoh 3-6:
Biaya masuk SMA favorit lima tahun mendatang diperkirakan Rp 30 juta. Untuk keperluan tersebut, seorang
ayah berniat membuka tabungan pendidikan yang memberikan bunga 18% per tahun. Berapa uang yang
harus disetorkan sekarang?
Jawab:
P = Rp 30 juta (1/(1 + 0,18)5)
= Rp 30 juta (P/F 18%, 5)
= Rp 30 juta (0,4371)= Rp 13.113.000

3.5.3. Uniform series compound-amount


Digunakan untuk menentukan nilai mendatang (F) dari suatu rangkaian pembayaran yang seragam (A)
yang terjadi pada setiap akhir periode tertentu dan tingkat bunga tertentu.

Contoh 3-7: F= ?
5 4 3
Seorang PNS menabung Rp 10 juta per tahun untuk keperluan investasi saat dia pensiun lima tahun yang
1 2 0
akan datang. Jika suku bunga yang diberikan bank 6% per tahun, berapa total uang yang dia kumpulkan
sampai akhir tahun ke-5?
Jawab: A
5
F = Rp 10 juta ((1 + 0,06) 1)/ 0,06
= Rp 10 juta (F/A6%, 5)
= Rp 10 juta (5,6371)= Rp 56.371.000

3.5.4. Sinking-fund deposit


Digunakan untuk menghitung besarnya pembayaran seri yang seragam pada tiap akhir tahun dari sejumlah
uang pada masa mendatang pada akhir periode tertentu dan tingkat suku bunga tertentu.

Contoh 3-8:
Jika kita ingin memiliki uang $ 1000 pada akhir tahun keenam, berapa uang yang harus kita depositokan
setiap tahunnya, jika suku bunga deposito itu 10%/tahun?
F= $1000
6 .. ..
Jawab:
1 .. 0
A = $1000 (0,1/((1 + 0,1)6 -1))
= $ 1000 (A/F10%,6)
= $ 1000 (0,12961)= $ 129,61 A

3.5.5. Capital-recovery

Yuli Kusumawati, Catatan Kuliah Ekonomi Mineral - 14


Digunakan untuk menentukan nilai aliran kas yang seragam setiap akhir tahun selama periode tertentu
dengan suku bunga tertentu dari nilai sekarang.

Contoh 3-9:
Untuk ekspansi bisnisnya seorang pengusaha meminjam uang di bank sebesar Rp 100 juta dengan suku
bunga 5% per tahun untuk jangka waktu 10 tahun. Berapa angsuran yang dibayar pengusaha tersebut
setiap tahunnya?
P=Rp 100 jt
Jawab:
0 1 .. .. .. 10
A = Rp 100 juta {(0,05(1+0,05)10) / ((1+0,05)10 1)}
= Rp 100 juta (A/P5%,10)
= Rp 100 juta (0,1295)= Rp 12.950.000/tahun A A A A A
A=?
3.5.6. Uniform series present-worth
Digunakan untuk menentukan nilai sekarang dari sejumlah pembayaran yang seragam setiap akhir tahun
selama periode tertentu dan suku bunga tertentu.

Contoh 3-10:
Sebuah dealer menawarkan mobil dengan uang muka Rp 10 juta dan sisanya dapat diangsur Rp 2 juta/bulan
selama 100 bulan. Bila bunga yang diberlakukan dealer tersebut adalah 1%/bulan, berapa harga mobil itu
jika dibayar tunai saat ini?
Jawab: P=?
Nilai sekarang dari total angsuran: 100
0 1 .. .. ..
P = Rp 2 juta (P/A )
1%,100

= Rp 2 juta (63,029)
= Rp 126.038.000 A A A A A
A=
Harga mobil tersebut = uang muka + total angsuran
2jt/bln
H = Rp 10 juta + Rp 126.038.000
= Rp 136.038.000

3.5.7. Uniform gradient-series


Digunakan untuk menghitung pembayaran yang naik atau turun secara konstan pada setiap periode.
A = B + g {(1/i)-(n/(1+i)n 1))}
A = B + g (A/G i,n )

Contoh 3-11:
Biaya operasi dan pemeliharaan suatu mesin produksi dari tahun ke-1 sampai tahun ke-5 berturut-turut
adalah Rp 5 juta, Rp 7,5 juta, Rp 10 juta, Rp 12,5 juta, dan Rp 15 juta. Berapa biaya tersebut per tahunnya
jika diketahui tingkat suku bunga adalah 20% per tahun.
Jawab:
A = B + g (A/G i,n)
= Rp 5 juta + Rp 2,5 juta (A/G )
20%,5

= Rp 5 juta + Rp 2,5 juta (1,6045)

Yuli Kusumawati, Catatan Kuliah Ekonomi Mineral - 15


= Rp 9.011.250
Sama dengan diagram aliran kas:

Tabel 3.2. Rumus aljabar dan konversi rumus bunga majemuk

Yuli Kusumawati, Catatan Kuliah Ekonomi Mineral - 16


4. INDIKATOR-INDIKATOR KELAYAKAN INVESTASI
Tujuan:
Mampu memahami konsep analisis kelayakan investasi tunggal.
Mampu menganalisis kelayakan investasi tunggal berdasarkan indikator NPV, ROR, PVR, BCR, dan
PBP

Kelayakan suatu investasi (proyek) dari aspek ekonomi dapat dianalisis berdasarkan manfaat (benefit) dan
biaya (cost). Suatu proyek dikatakan layak untuk dikerjakan jika manfaat yang diberikan lebih besar
daripada biaya yang dikeluarkan. Untuk menilai keekonomian suatu proyek perlu dilihat seluruh pengeluaran
dan pendapatan sepanjang umur proyek tersebut (life cycle analysis), bukan hanya dengan melihat biaya
per satuan produksi di tahun tertentu.

Indikator keuntungan yang digunakan dalam pengambilan keputusan seyogyanya mempunyai ciri-ciri:
1. Dapat dengan tepat dalam membandingkan dan mengelompokkan kesanggupan memberikan
keuntungan (profitability) dari kesempatan-kesempatan penanaman modal.
2. Mencerminkan nilai waktu dari modal perusahaan dan secara realistis merupakan masukan bagi
kebijaksanaan fiskal dari perusahaan, termasuk investasi kembali di masa yang akan datang.
3. Dapat menunjukkan keuntungan meskipun sekecil-kecilnya.
4. Mencakup pernyataan-pernyataan kuantitatif dari resiko.
5. Menggambarkan faktor-faktor lain, seperti hasil-hasil gabungan, resiko, dan kekayaan perusahaan bila
mungkin.

Beberapa indikator yang digunakan untuk menilai kelayakan proyek/investasi antara lain:
a. Net present value (NPV)
b. Rate of return (ROR)
c. Present value ratio (PVR) atau Benefit-cost ratio (BCR)
d. Payback period (PBP)

Pada dasarnya untuk analisis investasi tunggal, semua indikator tersebut akan memberikan hasil keputusan
yang konsisten satu sama lain, tetapi informasi spesifik yang dihasilkan tentu akan berbeda. Sehingga dalam
prakteknya beberapa indikator sering digunakan secara bersamaan untuk mendapatkan gambaran yang
lebih komprehensif.

4.1. Net present value (NPV)


NPV adalah perbedaan antara total present worth dari total penerimaan (revenue) atau penghematan
(saving) dan present worth dari pengeluaran sepanjang umur proyek pada discount rate yang diberikan.
NPV = PW revenue - PW cost
NPV menunjukkan keuntungan proyek secara absolut.Suatu proyek dikatakan layak (feasible) apabila
NPV>0.
Cara menghitung NPV bukan trial and error, memperhitungkan nilai waktu uang, menggunakan discount rate
sama dengan MARR, dan bisa mempertimbangkan resiko.

Contoh 4-1:
Suatu proyek memiliki aliran kas sebagai berikut:

Tahun ke- 0 1 2 3 4 5
Aliran kas (x Rp 1.000.000) -100 20 30 20 40 40
Analisis kelayakan proyek tersebut berdasarkan NPV jika diketahui MARR = 10%.
Jawab:
NPV = PW revenue - PW cost
= 20(P/F,10%,1)+30(P/F,10%,2)+20(P/F,10%,3)+40(P/F,10%,4)+40(P/F,10%,5)-100

Yuli Kusumawati, Catatan Kuliah Ekonomi Mineral - 17


= 20(0,9091)+30(0,8264)+20(0,7513)+40(0,6830)+40(0,6209)-100
= 10,156
Karena NPV>0, maka proyek tersebut layak.

4.2. Rate of return (ROR)


ROR adalah discount rate (i) yang menyebabkan NPV = 0.ROR merupakan laju perolehan per tahun yang
dihasilkan oleh investasi suatu proyek (menunjukkan keuntungan secara relatif terhadap skala investasi
proyek).
Cara menghitung ROR adalah coba-coba (trial and error), memperhitungkan nilai waktu uang, lebih
dipengaruhi cashflow awal, tidak tergantung nilai absolut cashflow, tidak bisa dihitung jika semua cashflow
negatif atau positif atau belum balik modal, dan bisa diperoleh nilai ganda.
Suatu proyek dikatakan layak jika ROR>MARR.

Contoh 4-2:
Suatu proyek memiliki aliran kas sebagai berikut:

Tahun ke- 0 1 2 3 4 5
Aliran kas (x Rp 1.000.000) -100 20 30 20 40 40

Analisis kelayakan proyek tersebut berdasarkan ROR jika diketahui MARR = 10%.
Jawab:
ROR adalah nilai i yang menghasilkan NPV=0, diperoleh dengan cara coba-coba:
Untuk i = 10% diperoleh NPV = 10,156 (perhitungannya seperti contoh 4.1)
Untuk i = 15%, nilai NPV adalah:

15% NPV = 20(P/F,15%,1)+30(P/F,15%,2)+20(P/F,15%,3)+40(P/F,15%,4)+40(P/F,15%,5)-100


10,156
= 20(0,8696)+30(0,7561)+20(0,6575)+40(0,5718)+40(0,4972)-100
= -4,015
Selanjutnya dicari nilai ROR dengan interpolasi:
0
ROR10-4,015 10% 15 10 ROR=?
=
10,1560 10,156(4,015)

ROR = 10% + (15%-10%)((10,156/(10,156+4,02))


=13,58%
Karena ROR>MARR, maka proyek tersebut layak.

4.3. Present value ratio (PVR)


PVR adalah perbandingan antara NPV dengan nilai absolut investasi yang dikeluarkan (PW net cashflow
negatif).


PW net CF
NPV
PVR=

Proyek dikatakan layak jika PVR>0.

Contoh 4-3:
Suatu proyek memiliki aliran kas sebagai berikut:

Yuli Kusumawati, Catatan Kuliah Ekonomi Mineral - 18


Tahun ke- 0 1 2 3 4 5
Aliran kas (x Rp 1.000.000) -100 20 30 20 40 40

Analisis kelayakan proyek tersebut berdasarkan PVR jika diketahui MARR = 10%.
Jawab:
PVR = NPV/PW net CF (-)
= 10,156/-100= 0,10
Karena PVR>0, maka proyek tersebut layak.

4.4. Benefit-cost ratio (BCR)


BCR adalah perbandingan antara discounted benefit (saving atau revenue) dengan investasi.
+


PW net CF
PW net CF
BCR=

Proyek dikatakan layak jika BCR>1.


Hubungan PVR dan BCR adalah:
BCR = PVR + 1 atau PVR = BCR -1

Contoh 4-4:
Suatu proyek memiliki aliran kas sebagai berikut:

Tahun ke- 0 1 2 3 4 5
Aliran kas (x Rp 1.000.000) -100 20 30 20 40 40

Analisis kelayakan proyek tersebut berdasarkan BCR jika diketahui MARR = 10%.
Jawab:
BCR = PW net CF(+)/PW net CF (-)
= 20(P/F,10%,1)+30(P/F,10%,2)+20(P/F,10%,3)+40(P/F,10%,4)+40(P/F,10%,5) / -100
= 110,156/100
= 1,10
Atau
BCR = PVR + 1
= 0,10 + 1 = 1,10
Karena BCR>1, maka proyek tersebut layak.

4.5. Payback period (PBP)


PBP adalah jangka waktu yang diperlukan untuk mengembalikan modal investasi yang ditanam (investasi
mencapai titik impas). Simple PBP adalah PBP dengan discount rate sama dengan nol (undiscounted).
Kelemahan PBP adalah tidak mempersoalkan keuntungan investasi.PBP dihitung dengan cara interpolasi
berdasarkan aliran kas kumulatif.Diterima atau tidaknya suatu proyek pada periode pengembalian tertentu
tergantung pada investor. Proyek akan diterima jika PBP lebih singkat dari periode terpendek yang
diinginkan investor.

Yuli Kusumawati, Catatan Kuliah Ekonomi Mineral - 19


Contoh 4-5:
Suatu proyek memiliki aliran kas sebagai berikut:

Tahun ke- 0 1 2 3 4 5
Aliran kas (x Rp 1.000.000) -100 20 30 20 40 40

Analisis kelayakan proyek tersebut berdasarkan undisconted dan discounted PBP jika diketahui MARR =
10%.
Jawab:
a. Simple PBP
Terlebih dulu menghitung aliran kas kumulatifnya:

Tahun ke- 0 1 2 3 4 5
Aliran kas (x Rp 1 juta) -100 20 30 20 40 40
Aliran kas kumulatif (x Rp 1 juta) -100 -80 -50 -30 10 50

Terlihat bahwa periode pengembalian investasi proyek tersebut antara tahun ke-3 dan ke-4, sehingga PBP
dapat dihitung dengan interpolasi:

terakhir

+

th berikutnya
CF
Cum .CF
terakhir +
PBP=tahun Cum .CF

30
PBP=3+
40 = 3 + 0,75= 3,75 tahun.

b. Discounted PBP dengan MARR 10%


Terlebih dulu menghitung aliran kas diskonto dan kumulatifnya:
DCF th-1 = 20(P/F,10%,1) = 20(0,9091) = 18,182
DCF th-2 = 30(P/F,10%,2) = 30(0,8264) = 24,792
DCF th-3 = 20(P/F,10%,3) = 20(0,7513) = 15,026
DCF th-4 = 40(P/F,10%,4) = 40(0,6830) = 27,320
DCF th-5 = 40(P/F,10%,5) = 40(0,6209) = 24,836

Tahun ke- 0 1 2 3 4 5
Discounted CF (x Rp 1 juta) -100 18,182 24,792 15,026 27,320 24,836
Cummulatif DCF (x Rp 1 juta) -100 -81,818 -57,026 -42,000 -14,680 10,156

Terlihat bahwa periode pengembalian investasi proyek tersebut antara tahun ke-4 dan ke-5, sehingga PBP
dapat dihitung dengan interpolasi:

Yuli Kusumawati, Catatan Kuliah Ekonomi Mineral - 20


terakhir

+

th berikutnya
CF
Cum .CF
terakhir +
PBP=tahun Cum .CF

14,680
PBP=4 +
24,836 = 4 + 0,59 = 4,59 tahun.

Yuli Kusumawati, Catatan Kuliah Ekonomi Mineral - 21


5. ANALISIS MUTUALLY EXCLUSIVE
Tujuan:
Mampu memahami konsep pemilihan alternatif investasi secara mutually exclusive.
Mampu melakukan analisis mutually exclusive berdasarkan indikator NPV, ROR, BCR, dan PVR.

Proyek dapat didefinisikan sebagai suatu kegiatan ekonomi dengan tujuan tertentu. Untuk mencapai tujuan
tersebut terdapat berbagai alternatif yang dapat dipilih. Pertimbangan untuk memilih alternatif yang terbaik
adalah yang membutuhkan biaya minimum atau yang menghasilkan keuntungan maksimum atau yang
waktu pengembalian investasinya paling singkat.

Secara umum investasi dibedakan menjadi dua, yaitu:


a. Investasi yang menghasilkan pendapatan (revenue) atau penghematan (saving). Pertimbangan yang
digunakan untuk menilai kelayakan investasi ini adalah keuntungan yang maksimum.
b. Investasi yang tidak menghasilkan revenue atau saving. Pertimbangan yang digunakan untuk menilai
kelayakan investasi ini adalah biaya yang minimum.

Analisis mutually exclusive adalah menganalisis beberapa alternatif dimana hanya ada satu alternatif yang
akan dipilih, yaitu alternatif yang akan menghasilkan pelayanan (service) terbaik, atau yang akan
memperbaiki, memperluas, mengembangkan operasi yang ada, atau yang akan menghasilkan pemasukan
(income) terbesar. Alternatif yang dipilih adalah yang mempunyaiROR, NPV, BCR, atau PVR yang paling
tinggi dibandingkan alternatif yang lain. Akan tetapi terkadang suatu alternatif mempunyai ROR yang lebih
tinggi dibanding alternatif yang lain tetapi NPV nya lebih rendah. Untuk kondisi tersebut maka indikator
kelayakan tersebut harus dihitung nilai inkrementalnya.

5.1. Analisis mutually exclusive alternatif dengan umur sama


Jika terdapat beberapa alternatif proyek/investasi yang mempunyai umur investasi yang sama, maka
prosedur analisis mutually exclusive alternatif tersebut adalah sebagai berikut:
1. Urutkan alternatif dari yang mempunyai investasi awal terendah ke tertinggi.
2. Buat cashflow inkremental (CF), yaitu alternatif yang investasi awalnya lebih tinggi
(penantang/challenger) dikurangi alternatif yang investasi awalnya lebih rendah (petahana/defender).
3. Tentukan nilai NPV inkremental, ROR incremental, BCR inkremental, PVR inkremental.
4. Analisis dengan kriteria sebagai berikut:
a. Jika NPV> 0, maka pilih alternatif dengan investasi yang lebih besar (penantang) dan sebaliknya.
b. Jika ROR> MARR, maka pilih alternatif dengan investasi yang lebih besar (penantang) dan
sebaliknya.
c. Jika BCR> 1, maka pilih alternatif dengan investasi yang lebih besar (penantang) dan sebaliknya.
d. Jika PVR> 0, maka pilih alternatif dengan investasi yang lebih besar (penantang) dan sebaliknya.

Contoh 5-1:
Untuk memenuhi kebutuhan listrik di area penambangan, investor bermaksud membeli generator listrik. Ada
tiga alternatif generator listrik yang ditawarkan oleh penjual, dengan biaya dan manfaat sebagai berikut:

Merek generator X Y Z
Harga (x Rp 1 juta) 17 25 28
Biaya operasi dan perawatan (x Rp 1 juta) 8 6 5
Nilai manfaat/tahun (x Rp 1 juta) 13 13 13
Nilai sisa (x Rp 1 juta) 5 8 14
Ketiga generator tersebut dibeli pada tahun ini (tahun ke-0) dan akan digunakan pada tahun depan (tahun
ke-1) serta memiliki umur pemakaian yang sama yaitu 5 tahun.Jika hanya satu generator yang akan dibeli,
tentukan generator mana yang paling menguntungkan dengan asumsi MARR 14%/tahun.
Jawab:

Yuli Kusumawati, Catatan Kuliah Ekonomi Mineral - 22


Diagram cash flow masing-masing alternatif adalah sebagai berikut:
Alternatif X:
OCX = 8 juta
OCX = 8 juta OCX = 8 juta IX = 13 juta
CX = 17 juta IX = 13 juta IX = 13 juta LX = 5 juta

0 1 ........................... 4 5
CFX0 = -17 juta CFX1 = 5 juta CFX4 = 5 juta CFX5 = 10 juta

Alternatif Y:
OCy = 6 juta
OCy = 6 juta OCy = 6 juta Iy = 13 juta
Cy = 25 juta Iy = 13 juta Iy = 13 juta Ly = 8 juta

0 1 ........................... 4 5
CFY0 = -25 juta CFY1 = 7 juta CFY4 = 7 juta CFY5 = 15 juta

Alternatif Z:
OCZ = 5 juta
OCZ = 5 juta OCZ = 5 juta IZ = 13 juta
CZ = 28 juta IZ = 13 juta IZ = 13 juta LZ = 14 juta

0 1 ........................... 4 5
CFZ0 = -28 juta CFZ1 = 8 juta CFZ4 = 8 juta CFZ5 = 22 juta

Kemudian urutkan alternatif dari yang mempunyai investasi terkecil sampai yang mempunyai investasi
terbesar, dengan cara membuat tabel cashflow masing-masing alternatif dan inkrementalnya:

Tahun ke- X Y Z Y-X Z-Y


0 -17 -25 -28 -8 -3
1 5 7 8 2 1
2 5 7 8 2 1
3 5 7 8 2 1
4 5 7 8 2 1
5 10 15 22 5 7

Selajutnya dianalisis berdasarkan nilai inkrementalnya:


Bandingkan generator X dengan generator Y:
NPVY-X @ 14% =- 8 + 2 (P/A,14%,4) + 5 (P/F,14%,5) = - 8 + 2(2,914) + 5(0,519) = 0,423 juta
Karena NPVY-X> 0, maka pilih Y (alternatif dengan investasi awal lebih besar).
Lalu bandingkan generator Y dengan generator Z:
NPVZ-Y @ 14% = - 3 + 1 (P/A,14%,4) + 7 (P/F,14%,5) = - 3 + 1(2,914) + 7(0,519) = 3,547 juta
Karena NPVZ-Y > 0, maka pilih Z (alternatif dengan investasi awal lebih besar).

RORY-X (dihitung dengan cara coba-coba) = 15,9%


Karena RORY-X> MARR, maka pilih Y (alternatif dengan investasi awal lebih besar).
Lalu bandingkan generator Y dengan generator Z:
RORZ-Y (dihitung dengan cara coba-coba) = 42,1%
Karena RORZ-Y> MARR, maka pilih Z (alternatif dengan investasi awal lebih besar).

PVRY-X @ 14% = (NPVY-X @ 14%)/PW net CFY-X (-) = 0,423 / 8 = 0,052


Karena PVRY-X> 0, maka pilih Y (alternatif dengan investasi awal lebih besar).
Lalu bandingkan generator Y dengan generator Z:
PVRZ-Y @ 14% = (NPVZ-Y @ 14%)/PW net CFZ-Y (-) =3,547 / 3 = 1,182
Karena PVRZ-Y> 0, maka pilih Z (alternatif dengan investasi lebih besar).

BCRY-X @ 14% = PVRY-X @ 14% + 1 = 0,052 + 1 = 1,052

Yuli Kusumawati, Catatan Kuliah Ekonomi Mineral - 23


Karena BCRY-X>1, maka pilih Y (alternatif dengan investasi awal lebih besar).
Lalu bandingkan generator Y dengan generator Z:
BCRZ-Y @ 14% = PVRZ-Y @ 14% + 1 = 1,182 + 1 = 2,182
Karena BCRZ-Y>1, maka pilih Z (alternatif dengan investasi awal lebih besar).

Hasil perhitungan dirangkum dalam tabel berikut ini:

Tahun ke- CF X CF Y CF Z CF (Y-X) CF (Z-Y)


0 -17 -25 -28 -8 -3
1 5 7 8 2 1
2 5 7 8 2 1
3 5 7 8 2 1
4 5 7 8 2 1
5 10 15 22 5 7
NPV 2,76 3,19 6,74 0,4 3,5
ROR 19,9% 18,6% 22,1% 15,9% 42,1%
PVR 0,16 0,13 0,24 0,05 1,18
BCR 1,16 1,13 1,24 1,05 2,18

Terbukti dengan menganalisis secara inkremental maka semua indikator tersebut memberikan hasil yang
sama/konsisten (generator Z paling layak untuk dipilih).

5.2. Analisis mutually exclusive alternatif dengan umur


berbeda
Seringkali alternatif-alternatif yang ada memiliki umur yang berbeda, sehingga diperlukan basis umur yang
sama untuk bisa menganalisisnya.
Perhatikan diagram cashflow dari dua alternatif berikut ini:
CA OCA1 OCA2

Alternatif A:
0 1 2 LA
CB OCB1 OCB2 OCB3
LB
Alternatif B:
0 1 2 3

Ada tiga metode untuk menyamakan basis umur alternatif, yaitu:


a. Berdasarkan kelipatan persekutuan terkecil dari masing-masing umur alternatif.
Kelemahan metode ini adalah jika diperoleh hasil kelipatan persekutuan terkecil yang besar maka akan
terlalu banyak dummy reinvestment (investasi ulang semu).
Alternatif A:
CA OCA1 OCA2 CA OCA1 OCA2 CA OCA1 OCA2
LA LA LA
0 1 2 2 3 4 4 5 6
Alternatif B:
CB OCB1 OCB2 OCB3 CB OCB1 OCB2 OCB3
b. Berdasarkan umur alternatif terpanjang LB LB
0Alternatif dengan
1 umur lebih pendek
2 3
akan ditambahkan
3 investasi ulang
4 semu sampai
5 periodenya6 sama
dengan alternatif dengan umur terpanjang.
Jika menggunakan alat yang sama maka nilai sisa pada investasi ulang semu adalah nilai buku pada
periode yang terpotong.
Jika menggunakan alat baru (CA CA) pada investasi ulang semu, maka nilai LA LA.
Alternatif A:
CA OCA1 OCA2 CA OCA3
LA LA
0 1 2 2 3

Yuli Kusumawati, Catatan Kuliah Ekonomi Mineral - 24


Alternatif B:
CB OCB1 OCB2 OCB3
LB
0 1 2 3

c. Berdasarkan umur alternatif terpendek.


Pada alternatif yang dikurangi umurnya, maka nilai sisa yang baru sama dengan nilai buku yang
terpotong (LB> LB).
Alternatif A:
CA OCA1 OCA2
LA
0 1 2
Alternatif B:
CA OCB1 OCB2
LB
0 1 2

Contoh 5-1:
Seorang investor dihadapkan pada pilihan untuk menjual bukit karst miliknya seharga Rp 150 juta atau
mengembangkan lahan tersebutdengan investasi Rp 300 juta pada tahun ke-0 dan Rp 400 juta pada tahun
ke-1 akan menghasilkan keuntungan tetap dari tahun ke-2 sampai tahun ke-10 sebesar Rp 200 juta. Jika
MARR yang ditetapkan adalah 15%, keputusan mana yang paling menguntungkan secara ekonomi?
Jawab:
Diagram cash flow masing-masing alternatif adalah:
Alternatif A (mengembangkan lahan):

-300 -400 200 200 200 200 200 200 200 200 200

0 1 2 3 4 5 6 7 8 9 10

Alternatif B (lahan dijual):

150 - - - - - - - - - -

0 1 2 3 4 5 6 7 8 9 10

Diagram inkremental cashflow (alternatif A alternatif B) adalah:

-450 -400 200 200 200 200 200 200 200 200 200

0 1 2 3 4 5 6 7 8 9 10

NPVA-B = - 450 - 400(P/F,15%,1) + 200(P/A,15%,9)(P/F,15%,1)


= - 450 400(0,870) + 200(4,772)(0,870)
= Rp 32,3 juta
Keputusan pilih alternatif A (NPVA-B> 0, pilih alternatif dengan investasi awal lebih besar)

20%
32,3 RORA-Bdihitung dengan cara coba-coba:
NPVA-B @15% = Rp 32,3 juta
NPVA-B @ 20% = - 450 - 400(P/F,20%,1) + 200(P/A,20%,9)(P/F,20%,1)

0 = - 450 - 400(0,833) + 200(4,031)(0,833)


=
15%Rp -111,6 juta
-111,6 RORA-B =?
Selanjutnya dicari nilai RORA-B dengan interpolasi:
(RORA-B-15%)/(32,3-0) = (20%-15%)/(32,3-(-111,6))
RORA-B = 15% + (20%-15%)((32,3/(32,3+111,6))
=16%
Karena RORA-B>MARR, maka pilih alternatif A.

Yuli Kusumawati, Catatan Kuliah Ekonomi Mineral - 25


PVRA-B = NPVA-B /PW net CF (-)
= 32,3 /(-450 + -400(P/F,15%,1)
= 0,04
Karena PVRA-B> 0, maka pilih alternatif A.

BCRA-B = PVRA-B + 1
= 0,04 + 1
= 1,04
Karena BCRA-B>1, maka pilih alternatif A.

Yuli Kusumawati, Catatan Kuliah Ekonomi Mineral - 26


6. ANALISIS NON-MUTUALLY EXCLUSIVE
Tujuan:
Mampu memahami konsep pemilihan alternatif investasi non-mutually exclusive.
Mampu melakukan analisis non-mutually exclusive berdasarkan indikator NPV, ROR, BCR, dan PVR.

Analisis non-mutually exclusive adalah analisis terhadap beberapa alternatif dimana dari beberapa alternatif
tersebut dapat dipilih lebih dari satu alternatif, sesuai dengan tersedianya modal atau anggaran untuk
investasi tersebut.
Tujuan dari analisis non-mutually exclusive adalah untuk memaksimalkan keuntungan kumulatif yang dapat
dihasilkan dari investasi (cummulative net value atau cummulative future worth profit) berdasarkan
kombinasi beberapa alternatif.
Aplikasi analisis non-mutually exclusive antara lain pemilihan alternatif riset dan pengembangan, pemilihan
metode eksplorasi, serta meranking prospek pengeboran pada industri minyak.

Ada dua teknik yang digunakan untuk meranking proyek non-muatually exclusive, yaitu:
a. Growth rate of return (growth ROR), yaitu suatu tingkat suku bunga majemuk dimana investasi tumbuh.
b. Analisis rasio, baik menggunakan PVR atau BCR.

Individual ROR dan individual NPV tidak bisa digunakan untuk meranking proyek non-mutually exclusive,
karena individual NPV maupun RORyang besar belum tentu mencerminkan alternatif terbaik.Analisis
menggunakan net value analysis (NPV,NAV,NFV) pada proyek non-mutually exclusive digunakan untuk
memilih kombinasi proyek yang menghasilkan nilai bersih kumulatif terbesar.

6.1. Analisis non-mutually exclusive dengan umur alternatif


sama
Contoh 6-1:
Seorang pengusaha dihadapkan pada empat alternatif pembelian mesin seperti berikut:

Mesin A Mesin B Mesin C


Biaya investasi (xRp 1000) 10.000 25.000 35.000
Keuntungan (xRp 1000) 6.000 10.000 15.000
Umur mesin 5 tahun 5 tahun 5 tahun

Jika pengusaha itu memiliki dana Rp 35 juta untuk pembelian mesin tersebut, dan dia juga mempunyai
pilihan untuk berinvestasi pada proyek lain yang memberi keuntungan 20%/tahun, mesin mana yang dipilih
secara non-mutually exclusive?
Jawab:
NPVA = -10 juta + 6 juta(P/A,20%,5) = -10 juta + 6 juta (2,991) = Rp 7,946 juta
NPVB = -25 juta + 10 juta(P/A,20%,5)= -25 juta + 10 juta (2,991) = Rp 4,910 juta
NPVC = -35 juta + 15 juta(P/a,20%,5) = -35 juta + 15 juta (2,991) = Rp 9,865 juta
Analisis:
Total harga mesin A + mesin B = Rp 35 juta, NPVA + NPVB= Rp 12,856 juta.
Harga mesin C = Rp 35 juta, NPVC = Rp 9,865 juta.
Kesimpulan: lebih menguntungkan membeli mesin A + mesin B.

PVRA = NPVA /PW net CFA (-) = 7,946 /-10 = 0,795


PVRB = NPVB /PW net CFB (-) = 4,910 /-25 = 0,196
PVRC = NPVC /PW net CFC (-) = 9,865 /-35 = 0,282

Yuli Kusumawati, Catatan Kuliah Ekonomi Mineral - 27


Berdasarkan PVR diperoleh ranking sebagai berikut:
Mesin A (PVRA = 0,795) paling menguntungkan, disusul mesin C (PVR C = 0,282), dan terakhir mesin B (PVR B
= 0,196).

BCRA = PVRA + 1 = 0,795 + 1= 1,795


BCRB = PVRB + 1 = 0,196+ 1 = 1,196
BCRC = PVRC + 1 = 0,282 + 1= 1,282
Berdasarkan BCR diperoleh ranking sebagai berikut:
Mesin A (BCRA= 1,795) paling menguntungkan, disusul mesin C (BCR C = 1,282), dan terakhir mesin B (BCR B
= 1,196).

Growth ROR mesin A:

-10jt 6jt 6jt 6jt 6jt 6jt


Initial
0 1 2 3 4 5
Keuntungan yang diperoleh diinvestasikan lagi ke proyek lain dengan i* = 20%:

- 6jt 6jt 6jt 6jt 6jt


Reinvest
FA = ?
0 1 2 3 4 5
Menghasilkan total pendapatan di akhir umur proyek sebesar:
FA = 6jt(F/A,20%,5) = 6jt(7,442) = Rp 44,65 juta

-10jt - - - - -
Initial +Reinvest FA = 44,65jt
0 1 2 3 4 5
40%
2,01 Growth ROR alternatif A (GRORA)dihitung dengan coba-coba:
NPVA @ 35% = - 10jt + 44,65jt (P/F,35%,5) = - 10jt + 44,65jt (0,269) = Rp 2,01 jt
NPVA @ 40% = - 10jt + 44,65jt (P/F,40%,5) = - 10jt + 44,65jt (0,186) = Rp -1,69 jt
Selanjutnya dicari
35%nilai interpolasinya: GROR =?
(GRORA-35%)/(2,01-0)
-1,69 = (40%-35%)/(2,01-(-1,69))
GRORA = 35% + (40%-35%)((2,01/(2,01+1,69))=37,7%

Growth ROR mesin B:

-25jt 10jt 10jt 10jt 10jt 10jt


Initial
0 1 2 3 4 5
Keuntungan yang diperoleh diinvestasikan lagi ke proyek lain dengan i* = 20%:

- 10jt 10jt 10jt 10jt 10jt


Reinvest
FB = ?
0 1 2 3 4 5
Menghasilkan total pendapatan di akhir umur proyek sebesar:
FB = 10jt(F/A,20%,5) = 10jt(7,442) = Rp 74,42 juta

-25jt - - - - -
Initial +Reinvest FB = 74,42jt
0 1 2 3 4 5
25%
4,92 Growth ROR alternatif B (GRORB)dihitung dengan coba-coba:
NPVB @ 20% = - 25jt + 74,42jt (P/F,20%,5) = - 25jt + 74,42jt (0,402) = Rp 4,92 jt
NPVB @ 25% = - 25jt + 74,42jt (P/F,25%,5) = - 25jt + 74,42jt (0,328) = Rp -0,59 jt
Selanjutnya dicari
20%nilai interpolasinya: GROR =?
(GRORB- 20%)/(4,92-0)
-0,59 = (25%-20%)/(4,92-(-0,59))

Yuli Kusumawati, Catatan Kuliah Ekonomi Mineral - 28


GRORB = 20% + (25%-20%)((4,92/(4,92+0,59)) = 24,46%

Growth ROR mesin C:

-35jt 15jt 15jt 15jt 15jt 15jt


Initial
0 1 2 3 4 5
Keuntungan yang diperoleh diinvestasikan lagi ke proyek lain dengan i* = 20%:

- 15jt 15jt 15jt 15jt 15jt


Reinvest
FC = ?
0 1 2 3 4 5
Menghasilkan total pendapatan di akhir umur proyek sebesar:
FC = 15jt(F/A,20%,5) = 15jt(7,442) = Rp 111,63 juta
-35jt - - - - -
FC = 111,63jt
Initial + Reinvest
0 1 2 3 4 5
30%
1,61
Growth ROR alternatif C (GRORC)dihitung dengan coba-coba:
NPVC @ 30% = - 35jt + 111,63jt (P/F,20%,5) = - 35jt + 111,63jt (0,269) = Rp -4,97 jt
NPVC @ 25% 25% = - 35jt + 111,63jt (P/F,25%,5) = - 35jt + 111,63jt (0,328) = Rp 1,61 jt
GROR =?
Selanjutnya dicari-4,97
nilai interpolasinya:
(GRORC- 25%)/(1,61-0) = (30%-25%)/(1,61-(-4,97))
GRORC = 25% + (30%-25%)((1,61/(1,61+4,97)) = 26,22%

Berdasarkan growth ROR diperoleh ranking sebagai berikut:


Mesin A (GRORA=37,7%) paling menguntungkan, lalu mesin C (GROR C=26,22%), terakhir mesin B
(GRORB=24,46%).
Hasil ini konsisten dengan ranking metode PVR dan BCR.

6.2. Analisis non-mutually exclusive dengan umur alternatif


berbeda
Contoh 6-2:
Seorang manager memiliki anggaran sebesar Rp 50 juta untuk dialokasikan pada alternatif investasi berikut
ini dengan pemilihan non-mutually exclusive. Tingkat pengembalian minimum yang ditetapkan adalah 10%.
Tentukan alternatif mana yang sebaiknya dipilih?

-50jt 20jt 20jt


A) L = 50jt
0 1 2

-30jt 10jt 10jt 10jt 10jt 10jt


B) L = 30jt
0 1 2 3 4 5

-20jt 5jt 5jt 5jt 5jt 5jt 5jt 5jt


C) L = 20jt
0 1 2 3 4 5 6 7

Jawab:
NPVA = -50 jt + 20 jt(P/A,10%,2) + 50jt(P/F,10%,2) = -50 jt + 20 jt(1,736) + 50jt(0,826) = 26,033 jt
NPVB = -30 jt + 10 jt(P/A,10%,5) + 30jt(P/F,10%,5) = -30 jt + 10 jt(3,791) + 30jt(0,621) = 26,535 jt
NPVC = -20 jt + 5 jt(P/A,!0%,5) + 20jt(P/F,10%,7) = -20 jt + 5 jt(4,868) + 20jt(0,513) = 14,605 jt
Analisis:
Total harga mesin B + mesin C = Rp 50 juta, NPVB + NPVC = Rp 41,140 juta.

Yuli Kusumawati, Catatan Kuliah Ekonomi Mineral - 29


Harga mesin A = Rp 50 juta, NPVA = Rp 26,033 juta.
Kesimpulan: lebih menguntungkan membeli mesin B + mesin C.

PVRA = NPVA /PW net CFA (-) = 26,033 /-50 = 0,52


PVRB = NPVB /PW net CFB (-) = 26,535 /-30 = 0,88
PVRC = NPVC /PW net CFC (-) = 14,605 /-20 = 0,73
Berdasarkan PVR diperoleh ranking sebagai berikut:
Mesin B (PVRB = 0,88) paling menguntungkan, disusul mesin C (PVR C = 0,73), dan terakhir mesin A (PVR A =
0,52).

BCRA = PVRA + 1 = 0,52 + 1 = 1,52


BCRB = PVRB + 1 = 0,88 + 1 = 1,88
BCRC = PVRC + 1 = 0,73 + 1 = 1,73
Berdasarkan BCR diperoleh ranking sebagai berikut:
Mesin B (BCRB = 1,88) paling menguntungkan, disusul mesin C (BCR C = 1,73), dan terakhir mesin A (BCR A =
1,52).

Growth ROR mesin A:


-50jt 20jt 20jt
Initial L = 50jt
0 1 2

Keuntungan yang diperoleh diinvestasikan lagi ke proyek lain dengan i* = 10%:


50jt
20jt 20jt - - - - -
Reinvest
F=?
0 1 2 3 4 5 6 7
Menghasilkan total pendapatan di akhir umur proyek sebesar:
FA = 20jt(F/P,10%,6) + 70jt(F/P,10%,5) = 20jt(1,772) + 70jt(1,611) = Rp 148,21 juta

Initial + -50jt - - - - - - -
Reinvest F= 148,21jt
20%
5,73 0 1 2 3 4 5 6 7

Growth ROR alternatif A (GRORA)dihitung dengan coba-coba:


NPVA @ 15% = - 50jt + 148,21jt (P/F,15%,7) = - 50jt + 148,21jt (0,376) = Rp 5,73 jt
NPVA @ 20% 15% = - 50jt + 148,21jt (P/F,20%,7) = - 50jt + 148,21jt (0,279) = Rp -8,65 jt
GROR =?
Selanjutnya dicari-8,65
nilai interpolasinya:
(GRORA-15%)/(5,73-0) = (20%-15%)/(5,73-(-8,65))
GRORA = 15% + (20%-15%)((5,73/(5,73+8,65)) = 17%

Growth ROR mesin B:

-30jt 10jt 10jt 10jt 10jt 10jt


Initial L = 30jt
0 1 2 3 4 5
Keuntungan yang diperoleh diinvestasikan lagi ke proyek lain dengan i* = 10%:
30jt
10jt 10jt 10jt 10jt 10jt - -
Reinvest F=?
0 1 2 3 4 5 6 7
Menghasilkan total pendapatan di akhir umur proyek sebesar:
FB = [10jt(F/A,10%,5) + 30jt](F/P,10%,2) = 10jt(6,105) + 30jt(0,1,21) = Rp 110,17 juta

-30jt - - - - - - -
F=110,17jt
0 1 2 3 4
Yuli Kusumawati, 5
Catatan Kuliah Mineral - 30
6 Ekonomi 7
Initial +
Reinvest

25% Growth ROR alternatif B (GRORB)dihitung dengan coba-coba:


0,74
NPVB @ 20% = - 30jt + 110,17jt (P/F,20%,7) = - 30jt + 110,17jt (0,279) = Rp 0,74 jt
NPVB @ 25% = - 30jt + 110,17jt (P/F,25%,7) = - 30jt + 110,17jt (0,210) = Rp -6,86 jt
Selanjutnya dicari
20%nilai interpolasinya: GROR =?
-6,86 = (25%-20%)/(0,74-(-6,86))
(GRORB- 20%)/(0,74-0)
GRORB = 20% + (25%-20%)((0,74/(0,74+6,86)) = 20,4%

Growth ROR mesin C:

-20jt 5jt 5jt 5jt 5jt 5jt 5jt 5jt


Initial L = 20jt
0 1 2 3 4 5 6 7
Keuntungan yang diperoleh diinvestasikan lagi ke proyek lain dengan i* = 10%:
20 jt
5jt 5jt 5jt 5jt 5jt 5jt 5jt
Reinvest F=?
0 1 2 3 4 5 6 7
Menghasilkan total pendapatan di akhir umur proyek sebesar:
FC = 5jt(F/A,10%,7) + 20jt = 5jt(9,487) + 20jt = Rp 67,43 juta

Initial + -20jt
Reinvest F = 67,43jt
20% 0 1 2 3 4 5 6 7
5,35
Growth ROR alternatif C (GRORC)dihitung dengan coba-coba:
NPVC @ 20% = - 20jt + 67,43jt (P/F,20%,7) = - 20jt + 67,43jt (0,279) = Rp -1,19 jt
NPVC @ 15% 15% = - 20jt + 67,43jt (P/F,15%,7) = - 20jt + 67,43jt (0,376) = Rp 5,35 jt
GROR =?
Selanjutnya dicari-1,19
nilai interpolasinya:
(GRORC- 15%)/(5,35-0) = (20%-15%)/(5,35-(-1,19))
GRORC = 15% + (20%-15%)((5,35/(5,35+1,19)) = 19,1%

Berdasarkan growth ROR diperoleh ranking sebagai berikut:


Mesin B (GRORB=20,4%) paling menguntungkan, disusul mesin C (GROR C=19,1%), terakhir mesin A
(GRORA=17%).
Hasil ini konsisten dengan ranking metode PVR dan BCR.

Yuli Kusumawati, Catatan Kuliah Ekonomi Mineral - 31


7. DEPRESIASI, DEPLESI, AMORTISASI
Tujuan:
Memahami konsep depresiasi, deplesi, dan amortisasi.
Mampu menghitung depresiasi, deplesi, dan amortisasi dengan metode yang sesuai.
Memahami pengaruh depresi, deplesi, dan amortisasi terhadap pajak.

7.1. Depresiasi
Depresiasi adalah penurunan dalam nilaifisik properti seiring dengan waktu danpenggunaannya.
Dalamkonsep akuntansi, depresiasi adalahpemotongan tahunan terhadappendapatan sebelum pajak
sehinggapengaruh waktu dan penggunaan atasnilai aset dapat terwakili dalam laporan keuangan
suatuperusahaan.
Faktor-faktor penyebab depresiasi:
Keausan atau penurunan performa mesin/alat akibat pemakaian (physical degradation).
Perkembangan teknologi yang menghasilkan mesin/alat yang lebih efisian dan ekonomis (functional
depreciation).

Syarat-syarat aset yang dapat didepresiasi:


Digunakan dalam usaha untuk menghasilkanpendapatan.
Mempunyai umur manfaattertentu.
Umur aset harus lebih dari satu tahun.
Nilai aset turun karena pemakaian.

Komponen dalam perhitungan depresiasi:


Nilai awal, yaitu biaya awal untuk mendapatkan aset (harga beli ditambah pajak), termasuk biaya
transportasi dan biaya lain sampai aset tersebut dapat digunakan sesuai fungsinya.
Nilai sisa, adalah perkiraan nilai aset pada akhir umur manfaatnya, merupakan harga jual suatu aset jika
tidak lagi digunakan untuk proses produksi.
Nilai buku, adalah nilai aset sesuai dengan laporan akuntansi yang mewakili jumlah modal yang masih
diinvestasikan pada aset tersebut. Sama dengan nilai awal (termasuk segala penyesuaian) dikurangi
dengan pengurangan karena depresiasi.
Tingkat/laju depresiasi, adalah prosentase depresiasi per tahun.
Masa manfaat, adalah perkiraan periode waktu pemakaian aset (properti) dalam kegiatan
produktif/menghasilkan pendapatan.

Tabel 7.1. Masa manfaat dan tarif penyusutan harta berwujud menurut UU No. 36 Tahun 2008

Keterangan: Ayat (1) menggunakan metode depresiasi straigth line, ayat (2) menggunakan metode
depresiasi double declining balance.

Yuli Kusumawati, Catatan Kuliah Ekonomi Mineral - 32


Metode perhitungan depresiasiantara lain:
Metode garis lurus (sraigth line)
Metode keseimbangan menurun ganda (double declining balance)
Metode jumlah digit tahun (sum of year digit)
Metode sinking fund
Metode unit produksi (production unit)

7.1.1. Straigth line (SL) depreciation


Metode ini mengalokasikan depresiasi aset secara konstan setiaptahunnya selama umur
manfaatnya.Dengan metode ini nilai buku (BV) pada akhir umur aset sama dengan nilai sisa (S).

Tingkat/laju depresiasi per tahun: d = 100% / N

Besarnya depresiasi dari tahun pertama sampai tahun ket adalah tetap:

Sedangkan nilai buku (book value) pada akhir tahun ke-t adalah:
Dimana:
Dt = depresiasi pada tahun ke-t (t = 1,2,...,n)
BVt = book value (nilai buku) aset
d = tingkat/laju depresiasi tahunan
P = nilai awal aset
S = salvage value (nilai sisa) aset
N = umur aset (dalam satuan tahun)
t = akhir tahun depresiasi yang ditinjau

Contoh 7-1:
Perusahaan ABC membeli satu unit mesin seharga $ 10,000. Diharapkanmasa pakai mesin tersebut selama
5 tahun.Pada akhir umur ekonomisnya mesin tersebut diperkirakan memiliki nilai sisa $ 1,000.
Hitung nilai depresiasi dengan metode straigth line dan nilai buku pada akhir tahun ke-1 sampai tahun ke-5
dari mesin tersebut.
Jawab:
P = $ 10,000
S = $ 1,000
N = 5 tahun
t = akhir tahun depresiasi yang ditinjau
Tingkat/laju depresiasi per tahun:
d = 100%/5 = 20%
Nilai depresiasi tiap tahun:
Dt = (P-S)/N
= ($ 10,000 $ 1,000)/5= $ 1,800
Nilai bukuakhir tahun ke-1 sampai ke-5:

BV1 = $ 10,000 $ 1,800 = $ 8,200


BV2 = $ 8,200 $ 1,800 = $ 6,400
BV3 = $ 6,400 $ 1,800 = $ 4,600
BV4 = $ 4,600 $ 1,800 = $ 2,800
BV5 = $ 2,800 $ 1,800 = $ 1,000

Besarnya depresiasi dan nilai buku disajikan pada tabel berikut:

Yuli Kusumawati, Catatan Kuliah Ekonomi Mineral - 33


7.1.2. Double declining balance (DDB) depreciation
Metode ini memakai tingkat/laju depresiasi dua kali (double) dari metode straigth line terhadap sisa nilai
buku.

Tingkat/laju depresiasi tahunan adalah: d = 200%/N

Besarnya depresiasi tahun ket adalah:

Sedangkan nilai buku (book value) pada akhir tahun ke-t adalah:

Dimana:
Dt = depresiasi pada tahun ke-t (t = 1,2,...,n)
BVt = book value (nilai buku) aset
d = tingkat/laju depresiasi tahunan
P = nilai awal aset
S = salvage value (nilai sisa) aset
N = umur aset (dalam satuan tahun)
t = akhir tahun depresiasi yang ditinjau

Contoh 7-2:
Perusahaan ABC membeli satu unit mesin seharga $ 10,000. Diharapkan masa pakai mesin tersebut selama
5 tahun. Pada akhir umur ekonomisnya mesin tersebut diperkirakan memiliki nilai sisa $ 1,000.
Hitung nilai depresiasi dengan metode declining balance dan nilai buku pada akhir tahun ke-1 sampai tahun
ke-5 dari mesin tersebut.
Jawab:
P = $ 10,000
S = $ 1,000
N = 5 tahun

Tingkat/laju depresiasi per tahun:


d = 200%/5 = 40%

Nilai depresiasi tahun ke-1 sampai ke-5:

D1 = 0,4 ($ 10,000 $ 0) = $ 4.000


D2 = 0,4 ($ 10,000 $ 4.000) = $ 2.400
D3 = 0,4 ($ 10,000 $ 6.400) = $ 1,440
D4 = 0,4 ($ 10,000 $ 7.840) = $ 864
D5 = 0,4 ($ 10,000 $ 8.704) = $ 518,4

Yuli Kusumawati, Catatan Kuliah Ekonomi Mineral - 34


Nilai buku akhir tahun ke-1 sampai ke-5:

BV1 = $ 10,000 $ 4.000 = $ 6.000


BV2 = $ 6.000 $ 2.400 = $ 3.600
BV3 = $ 3.600 $ 1.440 = $ 2.160
BV4 = $ 2.160 $ 864 = $ 1.296
BV5 = $ 1.296 $ 518,4 = $ 777,6

Besarnya depresiasi dan nilai buku disajikan pada tabel berikut:

7.1.3. Sum of years digits (SOYD) depreciation


Metode ini mengalokasikan beban depresiasi ast berdasarkan jumlah digit tahun pemakaiannya (SOYD),
sehingga alokasi biaya depresiasi tidak konstan. Dengan metode ini alokasi biaya depresiasi dibebankan
lebih besar pada awal-awal periode.

n(n+ 1)
Jumlah digit tahun adalah: SOYD=
2

n(t1)
Besarnya depresiasi tahun ket adalah: Dt= (PS )
SOYD

Sedangkan nilai buku (book value) pada akhir tahun ke-t adalah:
Dimana:
SOYD = jumlah digit tahun
Dt = depresiasi pada tahun ke-t (t = 1,2,...,n)
BVt = book value (nilai buku) aset
P = nilai awal aset
S = salvage value (nilai sisa) aset
N = umur aset (dalam satuan tahun)
t = akhir tahun depresiasi yang ditinjau

Contoh 7-3:
Perusahaan ABC membeli satu unit mesin seharga $ 10,000. Diharapkan masa pakai mesin tersebut selama
5 tahun. Pada akhir umur ekonomisnya mesin tersebut diperkirakan memiliki nilai sisa $ 1,000.
Hitung nilai depresiasi dengan metode sum of years digits (SOYD) dan nilai buku pada akhir tahun ke-1
sampai tahun ke-5 dari mesin tersebut.

Yuli Kusumawati, Catatan Kuliah Ekonomi Mineral - 35


Jawab:
P = $ 10,000
S = $ 1,000
N = 5 tahun

Jumlah digit tahun:

n(n+ 1) 5(5+1)
SOYD= = =15
2 2

Nilai depresiasi tiap tahun:

n(t1)
Dt= (PS )
SOYD

5(11) 5
D1= ( 10.0001.000 )= ( 9.000 )=3.000
15 15

5( 21) 4
D 2= ( 10.0001.000 ) = ( 9.000 )=2.400
15 15

5(31) 3
D3= ( 10.0001.000 )= ( 9.000 ) =1.800
15 15

5( 41) 2
D 4= ( 10.0001.000 ) = ( 9.000 )=1.200
15 15

5(51) 1
D5= ( 10.0001.000 )= ( 9.000 ) =600
15 15

Nilai buku pada akhir tahun ke-1 sampai ke-5:

BV1 = $ 10,000 $ 1,800 = $ 8,200


BV2 = $ 8,200 $ 1,800 = $ 6,400
BV3 = $ 6,400 $ 1,800 = $ 4,600
BV4 = $ 4,600 $ 1,800 = $ 2,800
BV5 = $ 2,800 $ 1,800 = $ 1,000

Besarnya depresiasi dan nilai buku disajikan pada tabel berikut:

Yuli Kusumawati, Catatan Kuliah Ekonomi Mineral - 36


7.1.4. Sinking fund depreciation
Metode depresiasi sinking fund membebankan depresiasi tahunan secara seragam dengan cara
mempertimbangkan faktor suku bunga, sehingga nilai aset yang akan didepresiasi dikalikan dengan equal
payment series sinking fund factor.
Besarnya depresiasi tahun ket adalah:
Sedangkan nilai buku (book value) pada akhir tahun ke-t adalah:
Dimana:
Dt = depresiasi pada tahun ke-t (t = 1,2,...,n)
BVt = book value (nilai buku) aset
P = nilai awal aset
S = salvage value (nilai sisa) aset
N = umur aset (dalam satuan tahun)
t = akhir tahun depresiasi yang ditinjau

Contoh 7-4:
Perusahaan ABC membeli satu unit mesin seharga $ 10,000. Diharapkan masa pakai mesin tersebut selama
5 tahun. Pada akhir umur ekonomisnya mesin tersebut diperkirakan memiliki nilai sisa $ 1,000. Hitung nilai
depresiasi dengan metode sinking fund dan nilai buku pada akhir tahun ke-1 sampai tahun ke-5 mesin
tersebut, jika i =20%.
Jawab:
P = $ 10,000
S = $ 1,000
N = 5 tahun
i = 20%

Nilai depresiasi tiap tahun:

D1 = ($ 10,000 $ 1.000)(A/F,20%,5) = $ 9.000 (0,1344) = $ 1.209,6


D2 = ($ 10,000 $ 1.000)(A/F,20%,5) = $ 9.000 (0,1344) = $ 1.209,6
D3 = ($ 10,000 $ 1.000)(A/F,20%,5) = $ 9.000 (0,1344) = $ 1.209,6
D4 = ($ 10,000 $ 1.000)(A/F,20%,5) = $ 9.000 (0,1344) = $ 1.209,6
D5 = ($ 10,000 $ 1.000)(A/F,20%,5) = $ 9.000 (0,1344) = $ 1.209,6

Nilai buku akhir tahun ke-1 sampai ke-5:

BV1 = $ 10,000 $ 1.209,6 = $ 8,790,4


BV2 = $ 6.000 $ 1.209,6 = $ 7.580,8
BV3 = $ 3.600 $ 1.209,6 = $ 6.371,2
BV4 = $ 2.160 $ 1.209,6 = $ 5.161,6
BV5 = $ 1.296 $ 1.209,6 = $ 3.952,0

Besarnya depresiasi dan nilai buku disajikan pada tabel berikut:

Yuli Kusumawati, Catatan Kuliah Ekonomi Mineral - 37


7.1.5. Production unit depreciation
Metode ini dipakai untuk peralatan yang menghasilkan layanan atau produksi berdasarkan kebutuhan per
unit produksi (misalnya, truk, excavator, mesin pompa air) dan tidak tergantung pada fungsi waktu.

Besarnya depresiasi tahun ket adalah: Dt = (Produksi/ Produksi) (P-S)

Sedangkan nilai buku (book value) pada tahun ke-t adalah: BVt = P(t-1) - Dt
Dimana:
Dt = depresiasi pada tahun ke-t (t = 1,2,...,n)
BVt = book value (nilai buku) aset
P = nilai awal aset
S = salvage value (nilai sisa) aset
t = akhir tahun depresiasi yang ditinjau

Contoh 7-5:
Perusahaan tambang pasir PT Galunggung membeli satu unit excavator seharga Rp 700 juta. Berdasarkan
spesifikasi teknis excavator tersebut mampu menambang pasir sebanyak 50.000 m 3 dan nilai sisa pada
akhir umur ekonomisnya diperkirakan Rp150 juta.
Perencanaan produksi pasir perusahaan ABC adalah sebagai berikut:
Tahun ke- Produksi (m3)
1 4.000
2 6.000
3 10.000
4 10.000
5 15.000
6 5.000
50.000

Hitung nilai depresiasi tahunan excavator tersebut dengan metode unit produksi.
Jawab:
P = Rp 700 juta
S = Rp 150 juta
Nilai depresiasi tahun ke-1 sampai tahun ke-6:
Dt = (Produksi/produksi)(P-S)
D1 = (4.000/50.000)(Rp 700 juta Rp 150 juta) = Rp 44 juta
D2 = (6.000/50.000)(Rp 700 juta Rp 150 juta) = Rp 66 juta
D3 = (10.000/50.000)(Rp 700 juta Rp 150 juta) = Rp 110 juta
D4 = (10.000/50.000)(Rp 700 juta Rp 150 juta) = Rp 110 juta
D5 = (15.000/50.000)(Rp 700 juta Rp 150 juta) = Rp 165 juta
D6 = (5.000/50.000)(Rp 700 juta Rp 150 juta) = Rp 55 juta

Yuli Kusumawati, Catatan Kuliah Ekonomi Mineral - 38


Nilai buku tahun ke-1 sampai ke-6:
BVt = P - Dt
BV1 = Rp 700 juta Rp 44 juta = Rp 656 juta
BV2 = Rp 656 juta Rp 66 juta = Rp 590 juta
BV3 = Rp 590 juta Rp 110 juta = Rp 480 juta
BV4 = Rp 480 juta Rp 110 juta = Rp 370 juta
BV5 = Rp 370 juta Rp 165 juta = Rp 205 juta
BV6 = Rp 205 juta Rp 55 juta = Rp 150 juta

Hasil hitungan depresiasi dan nilai buku disajikan pada tabel berikut:
Tahun ke- Produksi (m3) Depresiasi Nilai buku
1 4.000 Rp 44 juta Rp 656 juta
2 6.000 Rp 66 juta Rp 590 juta
3 10.000 Rp 110 juta Rp 480 juta
4 10.000 Rp 110 juta Rp 370 juta
5 15.000 Rp 165 juta Rp 205 juta
6 5.000 Rp 55 juta Rp 150 juta
50.000

7.2. Deplesi
Sumber daya alam juga termasuk aktiva tetap. Seperti halnya aktiva tetap lainnya, sumber daya alam
setelah dieksploitasi kemudian diolah terus-menerus maka lama kelamaan sumber daya alam akan habis.
Oleh karena itu perlu ada pembebanan biaya yang mirip dengan persediaan.
Deplesi adalah suatu bentuk potongan pajak atas berkurangnya sumberdaya alam (cadangan mineral atau
kekayaan hutan) yang diberikan selama umur ekonomis cadangan tersebut. Perhitungan deplesi dilakukan
setiap tahun.

Perbedaan depresiasi dan deplesi adalah sebagai berikut:


Deplesi merupakan penyusutan terhadap pengurangan kuantitatif yang terjadi dalam sumber daya
alam, sedangkan depresiasi merupakan penyusutan terhadap pengurangan service (manfaat ekonomi)
yang terjadi dalam aktiva tetap.
Deplesi digunakan untuk aktiva tetap yang tidak dapat diganti langsung dengan aktiva yang sama jika
sudah habis, sedangkan depresiasi digunakan untuk aktiva tetap yang pada umumnya dapat diganti jika
sudah habis.
Deplesi adalah pengakuan terhadap perubahan langsung dari suatu sumber alam menjadi barang yang
dapat dijual, sedangkan depresiasi adalah alokasi harga perolehan ke pendapatan periode yang
bersangkutan untuk suatu service yang dihasilkan (kecuali dalam perusahaan di mana depresiasi
dihitung berdasar hasil produksi).

Dalam menghitung besarnya deplesi perlu memperhatikan faktor-faktor:


Harga perolehan aktiva,
Taksiran nilai sisa jika sumber alam sudah selesai di eksploitasi
Taksiran hasil yang secara ekonomis dapat dieksploitasi.

Ada dua metode perhitungan deplesi untuk mineral, minyak, dan gas, yaitu:
a. Cost depletion
b. Percentage depletion
Sedangkan untuk menghitung deplesi hasil hutan biasanya menggunakan metode cost depletion.

Prosedur menentukan metode deplesi:


% deplesi
Hasil yang lebih kecil merupakan percentage depletiondeplesi
yang diijinkan
Pilih
Yuli metode yang menghasilkan
Kusumawati, Catatan Mineral - 39
pengurangan
Kuliah Ekonomi pajak lebih besar.

Batas % deplesi:
50% untuk mineral Cost depletion
100% untuk minyak dan gas
7.2.1. Cost depletion
Basis perhitungan cost depletion untuk proyek minyak dan pertambangan adalah:
Hak penguasaan.
Bonus penyewaan.
Biaya survey geologi/geofisika.
Biaya penaksiran dan legalitas.
Biaya-biaya sejenis yang diketahui dengan pasti.
Biaya eksplorasi penambangan ataupun biaya pengeboran minyak yang tidak nyata biasanya jarang
digunakan sebagai basis perhitungan cost depletion.

Mineral yang diambil dan dijual selama tahunberjalan


Cost depletion=Basis penyesuaian x
Mineral yang ditemukan pada awal tahun

Dimana:
Basis penyesuaian = Basis biaya Penyesuaian Deplesi kumulatif.

Contoh 7-6:
Suatu perusahaan mempunyai lahan pertambangan emas dengan cadangan diperkirakan 1.000.000ounces.
Biaya untuk mendapatkan IUP yang dikeluarkan pada tahun ke-0 adalah $150.000. Penambanganemas
diproyeksikan mulai tahun ke-1 dengan volume tetap sebesar 50.000 ounces/tahun.Hitunglah cost depletion
tahun ke-1 dan tahun ke-2.
Jawab:
Cost depletion tahun ke-1 = ($150.000)(50.000 ounces / 1.000.000 ounces) = $7.500
Cost depletion tahun ke-2 = ($150.000-$7.500)(50.000 ounces / 950.000 ounces) = $7.500

7.2.2. Percentage depletion


Percentage depletion adalah persentase tertentu dari pendapatan bersih (pendapatan kotor setelah
dikurangi royalti) dari penjualan mineral atau migas yang diambil dari lahan pertambangan selama tahun
pajak.
Persentase deplesi yang diijinkan biasanya tergantung pada jenis bahan tambang dan kapasitas produksi
perusahaan.

Contoh 7-7:
Suatu perusahaan mempunyai lahan pertambangan emas dengan cadangan diperkirakan 1.000.000 ounces.
Biaya untuk mendapatkan IUP yang dikeluarkan pada tahun ke-0 adalah $150.000. Penambanganemas
diproyeksikan mulai tahun ke-1 dengan volume tetap sebesar 50.000 ounces/tahun dengan harga jual
$29/ounce setelah royalti.Diasumsikan tingkat deplesi untuk emasadalah 15% dari pendapatan
bersih.Hitunglah percentage depletion tahun ke-1 dan tahun ke-2.
Jawab:
Percentage depletion tahun ke-1= 15% (50.000 ounce x $29) = $217.000
Percentage depletion tahun ke-2= 15% (50.000 ounce x $29) = $217.000

Yuli Kusumawati, Catatan Kuliah Ekonomi Mineral - 40


7.2.3. Memilih metode deplesi
Contoh 7-8:
PT Agung Linuwih mempunyai lahan emas dengan biaya hak penguasaan $150.000. Cadangan emas yang
ditemukan diperkirakan satu juta ounces. Produksi diproyeksikan dimulai tahun ke-1 sebesar 50.000
ounces/tahun dan dijual dengan harga $29/ouces setelah royalti. Biaya operasi dan overhead tahun ini
$180.000, dan depresiasi aset adalah $120.000/tahun. Diasumsikan tingkat produksi, biaya operasi, dan
harga jual akan sama pada tahun depan. Tingkat deplesi untuk emas adalah 15% dengan batas persentase
deplesi untuk perusahaan tersebut adalah 100%, pajak pertambangan $30.000/tahun, dan pajak
penghasilan 40%.
Jika perusahaan boleh memilih metode deplesi untuk mengurangi pajaknya, bandingkan besarnya
percentage depletion dan cost depletion tahun ke-1 dan tahun ke-2.
Jawab:
Tahun ke-1
Pendapatan bersih (50.000 ounces x $29) $1.450.000
- Biaya operasi -180.000
- Pajak pertambangan -30.000
- Depresiasi -120.000
Penghasilan kena pajak sebelum deplesi $1.120.000
- Batas persentase deplesi (1,0)($1.200.000) 1.120.000
- Persentage depletion(0,15)($1.450.000) -217.500*)
- Cost depletion 7.500**)
Penghasilan kena pajak $902.500
- Pajak 40% -361.000
Penghasilan bersih (keuntungan) 541.500
+ Depresiasi 120.000
+ Deplesi yang diambil 217.500
Cash flow penjualan tahun ke-1 $879.000

Keterangan:
*)Karena hasil percentage depletion 15% adalah $217.500 lebih kecil daripada batas 100% percetage
depletion, maka diperbolehkan menggunakan metode persentase deplesi. Pada tahun ke-2 jika pendapatan
dan pengurangan diasumsikan sama seperti tahun ke-1, maka batas percentage depletion juga sama,
sehingga diperbolehkan menggunakan metode persentase deplesi.
**)Cost depletion tahun ke-1 = ($150.000)(50.000 ounces / 1.000.000 ounces) = $7.500, lebih kecil daripada
percentage depletion sehingga perusahaan memilih percentage depletion untuk pengurangan deplesinya.
Basis biaya untuk perhitungan cost depletion tahun ke-2 harus disesuaikan untuk pengurangan aktual
deplesi yang diambil.
Cost depletion tahun ke-2 = ($150.000-$217.500)(50.000/950.000) < 0.
Sehingga pada tahun ke-2 metode cost depletion juga tidak digunakan untuk pengurangan deplesi. Dengan
kata lain percentage depletion digunakan lagi untuk menghitung deplesi pada tahun ke-2.

7.3. Amortisasi
Amortisasi adalah proses penyusutan untuk aset tidak berwujud (intangible) dan digunakan untuk
pengurangan pajak. Menurut pajak, harga perolehan aset harus diamortisasi jika aset tersebut digunakan
untuk mendapatkan, menagih, dan memelihara penghasilan dan memiliki masa manfaat lebih dari satu
tahun.
Harga aset atau biaya yang bisa diamortisasi misalnya biaya untuk pengembangan (development) tambang,
intangible cost pada pengeboran migas, hak patent, franchaise, dsb.

Yuli Kusumawati, Catatan Kuliah Ekonomi Mineral - 41


Cara mengitung amortisasi sama seperti perhitungan depresiasi garis lurus ataupun declining balance,
hanya saja amortisasi diberikan dengan tingkat (rate) tertentu dan selama periode tertentu, tergantung
kebijakan pajak masing-masing negara. Di Indonesia tarif amortisasi ditetapkan dalam pasal 11 A (2) UU
Pajak Penghasilan (PPh) No.36 Th 2008, seperti yang disajikan pada Tabel 7.1.

Tabel 7.1. Tarif Amortisasi berdasarkan UU No. 36 Tahun 2008


Masa manfaat Tarif amortisasi
Kelompok harta
(tahun) Garis lurus Saldo menurun
Kelompok 1 4 25% 50%
Kelompok 2 8 12,5% 25%
Kelompok 3 16 6,25% 12,5%
Kelompok 4 20 5% 10%

Jika biaya pengembangan seluruhnya (100%) digunakan untuk pengurangan pendapatan sebelum pajak,
maka tidak ada amortisasi.

Contoh 7-9:
Seorang investor akan mengembangkan suatu pertambangan. Biaya pengembangan sebesar $1 juta yang
diadakan pada bulan juli (bulan ke-7) tahun ini (tahun ke-0). Jika amortisasi yang diijinkan adalah 30% dari
biaya pengembangan dan periode amortisasi di atas 60 bulan, hitunglah pengurangan pajak yang diijinkan.
Jawab:
Basis amortisasi = 30% x $1.000.000 = $300.000
Jangka waktu dari bulan ke-7 sampai bulan ke-12 tahun ke-0 adalah 6 bulan, sehingga amortisasi yang
diambil pada tahun ke-0 adalah 6/60, tahun ke-1 sampai ke-4 adalah 12/60, dan tahun ke-5 adalah 6/60.
Tahun ke- Pengurangan amortisasi
1 (6/60) ($300.000) = $30.000
1-4 (12/60) ($300.000) = $60.000
5 (6/60) ($300.000) = $30.000

Yuli Kusumawati, Catatan Kuliah Ekonomi Mineral - 42


8. PENGARUH PAJAK DALAM ANALISIS LIFE CYCLE
COSTING
Tujuan:
Memahami konsep pajak.
Mampu menyusun cash flow setelah pajak
Mampu memahami pengaruh pajak dalam analisis life cycle costing

8.1. Pengertian pajak


Pajak (tax) adalah salah satu sumber keuangan pemerintah yang berasal dari rakyat.
Kegunaan pajak antara lain:
Membiayai kegiatan mengelola negara (membayar gaji PNS, subsidi, pembangunan fasilitas umum, dll).
Pemerataan pendapatan (orang yang memiliki pendapatan lebih besar akan dikenakan pajak lebih
banyak dibandingkan orang dengan penghasilan lebih kecil).
Mengurangi pemakaian barang yang secara umum tidak bermanfaat (misalnya: miras, rokok).
Memproteksi industri nasional, industri kecil, industri ramah lingkungan.

Beberapa jenis pajak yang dikenakan pada perusahaan antara lain:


Pajak pendapatan, yaitu pajak yang dipungut dari pendapatan usaha atau perorangan yang besarnya
dihitung berdasarkan persentase tertentu dari pendapatan bersih.
Pajak kekayaan, yaitu pajak atas kepemilikan tanah, bangunan, kendaraan, mesin/peralatan, dsb.
Pajak penjualan, yaitu pajak yang dipungut atas penjualan barang atau pemberian pelayanan dan tidak
ada kaitannya dengan pendapatan bersih atau keuntungan perusahaan.

Dalam industri pertambangan dikenal adanya pungutan yang bisa digunakan sebagai
pengurang/potongan(deduction) pendapatan sebelum pajak, antara lain:
Royalti, yaitu suatu bentuk kompensasi atas diberikannya hak istimewa untuk mengembangkan atau
memproduksi mineral atau migas. Kompensasi berupa pembagian keuntungan dari nilai mineral yang
diekstraksi.
Severance tax (mining tax), yaitu pajak negara atas pengambilan sumberdaya alam yang
perhitungannya berdasarkan nilai/jumlah produksi aktual.
Property tax (ad valorem tax), yaitu retribusi yang dipungut oleh otoritas perpajakan atas kekayaan baik
yang berupa barang bergerak maupun tidak bergerak.

Di samping itu potongan pajak juga berasal dari penyusutan atau penurunan nilai sebuah aktiva yang
memilki umur manfaat. Penyusutan adalah biaya non-kas yang berpengaruh terhadap pajak pendapatan,
karena merupakan potongan tahunan terhadap pendapatan sebelum pajak sehingga pengaruh waktu dan
penggunaan atas nilai aset dapat terwakili dalam laporan keuangan suatu perusahaan.
Jenis-jenis penyusutandalam industri pertambangan yaitu:
a. Depresiasi digunakan untuk aktiva tetap.
b. Amortisasi digunakan untuk aktiva tidak berwujud.
c. Deplesi digunakan untuk aktiva sumber daya Alam.

Dari sudut kepentingan pajak investor, metode tercepat dari pengurangan biaya adalah untuk mengeluarkan
atau mengurangi pajak secara penuh pada tahun dikeluarkan. Hal ini disebabkan semakin cepat diperoleh
pemotongan pajak, semakin cepat diperoleh keuntungan dari pemotongan tersebut sehingga akan
memperbaiki ekonomi proyek. Akan tetapi hukum pajak seringkali tidak mengijinkan hal yang demikian.

8.2. Cashflow setelah pajak


Contoh 8-1:

Yuli Kusumawati, Catatan Kuliah Ekonomi Mineral - 43


PT Agung Linuwih bermaksud menguasai dan mengembangkan lahan pertambangan yang diyakini
mengandung 500.000 ounces emas. Biaya hak penguasaan tambang tersebut $900.000, biaya
pengembangan $1.200.000, biaya pembelian peralatan tambang $1.000.000, dan biaya modal kerja $
300.000, semuanya dikeluarkan pada awal tahun ke-0. Umur pemakaian peralatan adalah 5 tahun, dan
depresiasi peralatan dihitung dengan metode straigth line dengan nilai sisa pada akhir umur pemakaian
peralatan adalah nol.
Produksi emas diproyeksikan dimulai tahun ke-1 sebesar 100.000 ounces/tahun selama 5 tahun.Harga jual
emas pada tahun ke-1 adalah $30/ouces, dan diperkirakan mengalami kenaikan 10%/tahun. Biaya operasi
tahun ke-1 adalah $1.000.000, dan mengalami kenaikan 8%/tahun.Diasumsikan tingkat produksi, biaya
operasi, dan harga jual akan sama pada tahun depan.
Royalti yang harus dibayarkan adalah 15% dari pendapatan (revenue). Pajak pertambangan $30.000/tahun,
dan pajak penghasilan 30%. Perusahaan boleh memilih metode cost deplesi ataupun persentase deplesi
untuk mengurangi pajaknyadengan tingkat deplesi adalah 15% dan batas persentase deplesi untuk
perusahaan tersebut adalah 50%.
Hitunglah:
a. Cashflow sebelum pajak
b. Cashflow setelah pajak jika 100% development cost digunakan untuk pengurang pajak dan deplesi
dihitung dengan metode cost depletion.
c. Cashflow setelah pajak jika 100% development cost digunakan untuk pengurang pajak dan deplesi
dihitung dengan metode percentage depletion.
d. Cashlow setelah pajak jika tingkat amortisasi 30% selama 5 tahun dan deplesi dihitung dengan metode
cost depletion.
e. Cashlow setelah pajak jika tingkat amortisasi 30% selama 5 tahun dan deplesi dihitung dengan metode
percentage depletion.
Diasumsikan negatif taxable income sebagai saving (expensed against other income).
Jawab:

Tahun ke- 0 1 2 3 4 5
Produksi (ounces/th) - 100.000 100.000 100.000 100.000 100.000
Harga jual ($/ounce) - 30 33 36,30 39,93 43,92
Biaya produksi ($/th) - 1.000.000 1.080.000 1.166.000 1.260.000 1.360.000

a. Cashflow sebelum pajak (x$1000):

Tahun ke- 0 1 2 3 4 5
Revenue 0 3.000 3.300 3.630 3.993 4.392
-Royalti (15%) 0 -450 -495 -545 -599 -659
Net revenue 0 2.550 2.805 3.086 3.394 3.733
-Operating cost 0 -1.000 -1.080 -1.166 -1.260 -1.360
Net income 0 1.550 1.725 1.919 2.134 2.373
+Working capital return 0 0 0 0 0 300
-Capital cost1) -3.400 0 0 0 0 0
CASH FLOW -3.400 1.550 1.725 1.919 2.134 2.673

Keterangan:
1)
Capital cost = working capital+acquisition rigths +depreciable equipment+development.
= $300 + $900 + $1.000 + $1.200 = $3.400

b. Cashflow setelah pajak jika 100% development cost digunakan untuk pengurang pajak dan deplesi
dihitung dengan metode cost depletion.

Tahun ke- 0 1 2 3 4 5
Revenue 0 3.000 3.300 3.630 3.993 4.392
-Royalty (15%) 0 -450 -495 -545 -599 -659
Net revenue 0 2.550 2.805 3.086 3.394 3.733

Yuli Kusumawati, Catatan Kuliah Ekonomi Mineral - 44


-Operating cost 0 -1.000 -1.080 -1.166 -1.260 -1.360
-Mining tax 0 -30 -30 -30 -30 -30
-Development cost -1.200 0 0 0 0 0
-Depreciation1) 0 -200 -200 -200 -200 -200
-Cost depletion2) 0 -180 -180 -180 -180 -180
Taxable income -1.200 1.140 1.315 1.510 1.724 1.963
-Tax (30%)3) 360 -342 -394,5 -453 -517,2 -588,9
Net income -840 798 920,5 1.057 1.206,8 1.374,1
+Depreciation 0 200 200 200 200 200
+Depletion 0 180 180 180 180 180
+Working capital return 0 0 0 0 0 300
-Capital cost4) -2.200 0 0 0 0 0
CASH FLOW -3.040 1.178 1.300,5 1.437 1.586,8 2.054,1
Keterangan:
1)
Deprecition tahun ke-n = (P-S)/n
= (1.000.000 0)/5 = $200.000 (tetap selama 5 tahun)

Mineral yang diambil dan dijual selama tahunberjalan


2 Cost depletion=Basis penyesuaian x
Mineral yang ditemukan pada awal tahun

Cost depletion th-1 = ($900.000)(100.000 ounces / 500.000 ounces) = $180.000


Cost depletion th-2 = ($900.000-$180.000)(100.000 ounces / 400.000 ounces) = $180.000
Cost depletion th-3 = ($720.000-$180.000)(100.000 ounces / 300.000 ounces) = $180.000
Cost depletion th-4 = ($540.000-$180.000)(100.000 ounces / 200.000 ounces) = $180.000
Cost depletion th-5 = ($360.000-$180.000)(100.000 ounces / 100.000 ounces) = $180.000
4)
Tax (30%), jika taxable income negatif maka dianggap sebagai saving.
5)
Capital cost = working capital+acquisition rigths +depreciable equipment.
= $300.000 + $900.000 + $1.000.000 = $2.200.000

c. Cashflow setelah pajak jika 100% development cost digunakan untuk pengurang pajak dan deplesi
dihitung dengan metode percentage depletion.

Tahun ke- 0 1 2 3 4 5
Revenue 0 3.000 3.300 3.630 3.993 4.392
-Royalty (15%) 0 -450 -495 -545 -599 -659
Net revenue 0 2.550 2.805 3.086 3.394 3.733
-Operating cost 0 -1.000 -1.080 -1.166 -1.260 -1.360
-Mining tax 0 -30 -30 -30 -30 -30
-Development cost -1.200 0 0 0 0 0
-Depreciation1) 0 -200 -200 -200 -200 -200
-Percentage depletion2) 0 -382 -421 -463 -509 -560
Taxable income -1.200 938 1.074 1.227 1.395 1.583
-Tax (30%)3) 360 -281,4 -322,2 -368,1 -418,5 -474,9
Net income -840 656,6 751,8 858,9 976,5 1.108,1
+Depreciation 0 200 200 200 200 200
+Depletion 0 382 421 463 509 560
+Working capital return 0 0 0 0 0 300
-Capital cost4) -2.200 0 0 0 0 0
CASH FLOW -3.040 1.238,6 1.372,8 1.521,9 1.685,5 2.168,1
Keterangan:
1)
Deprecition tahun ke-n = (P-S)/n
= (1.000.000 0)/5 = $200.000 (tetap selama 5 tahun)
2)
Percentage depletion diperbolehkan jika nilainya lebih kecil dari batas deplesi maksimal:
Batas deplesi maksimal = 50% x taxable income before depletion
Batas deplesi th-1 = 50%(2.550.000 - 1.000.000 30.000 0 200.000) = $660

Yuli Kusumawati, Catatan Kuliah Ekonomi Mineral - 45


Batas deplesi th-2 = 50%(2.805.000 - 1.080.000 30.000 0 200.000) = $747,5
Batas deplesi th-3 = 50%(3.086.000 - 1.166.000 30.000 0 200.000) = $845
Batas deplesi th-4 = 50%(3.394.000 - 1.260.000 30.000 0 200.000) = $952
Batas deplesi th-5 = 50%(3.733.000 - 1.360.000 30.000 0 200.000) = $1.071,5

Percentage depletion = 15% x net revenue


Percentage depletion th-1 = 15% x 2.550.000 = $382.000
Percentage depletion th-2 = 15% x 2.805.000 = $421.000
Percentage depletion th-3 = 15% x 3.086.000 = $463.000
Percentage depletion th-4 = 15% x 3.394.000 = $509.000
Percentage depletion th-5 = 15% x 3.733.000 = $560.000
3)
Tax (30%), jika taxable income negatif maka dianggap sebagai saving.
4)
Capital cost = working capital+acquisition rigths +depreciable equipment.
= $300.000 + $900.000 + $1.000.000 = $2.200.000

d. Cashflow setelah pajak jika amortisasi 30% selama 5 tahun dan deplesi dihitung dengan metode cost
depletion.

Tahun ke- 0 1 2 3 4 5
Revenue 0 3.000 3.300 3.630 3.993 4.392
-Royalty (15%) 0 -450 -495 -545 -599 -659
Net revenue 0 2.550 2.805 3.086 3.394 3.733
-Operating cost 0 -1.000 -1.080 -1.166 -1.260 -1.360
-Mining tax 0 -30 -30 -30 -30 -30
-Development cost1) -840 0 0 0 0 0
-Amortization2) -72 -72 -72 -72 -72 0
-Depreciation3) 0 -200 -200 -200 -200 -200
-Cost depletion4) 0 -180 -180 -180 -180 -180
Taxable income -912 1.068 1.243 1.438 1.652 1.963
-Tax (30%)5) 273,6 -320,4 -372,9 -431,4 -495,6 -588,9
Net income -638,4 747,6 870,1 1.006,6 1.156,4 1.374,1
+Amortization 72 72 72 72 72 0
+Depreciation 0 200 200 200 200 200
+Depletion 0 180 180 180 180 180
+Working capital return 0 0 0 0 0 300
-Capital cost6) -2.560 0 0 0 0 0
CASH FLOW -3.126,4 1.199,6 1.322,1 1.458,6 1.608,4 2.054,1

Keterangan:
1)
Development cost = 70% x $1.200 = $840
2)
Amortisasi tahun ke-n = 30% (dev. cost)/periode amortisasi
= 30% (1.200.000)/5 = $72.000 (tetap sampai tahun ke-5)
3)
Deprecition tahun ke-n = (P-S)/n
= (1.000.000 0)/5 = $200.000 (tetap selama 5 tahun)

Mineral yang diambil dan dijual selama tahunberjalan


4 Cost depletion=Basis penyesuaian x
Mineral yang ditemukan pada awal tahun

Cost depletion th-1 = ($900.000)(100.000 ounces / 500.000 ounces) = $180.000


Cost depletion th-2 = ($900.000-$180.000)(100.000 ounces / 400.000 ounces) = $180.000
Cost depletion th-3 = ($720.000-$180.000)(100.000 ounces / 300.000 ounces) = $180.000
Cost depletion th-4 = ($540.000-$180.000)(100.000 ounces / 200.000 ounces) = $180.000
Cost depletion th-5 = ($360.000-$180.000)(100.000 ounces / 100.000 ounces) = $180.000

Yuli Kusumawati, Catatan Kuliah Ekonomi Mineral - 46


5)
Tax (30%), jika taxable income negatif maka dianggap sebagai saving.
6)
Capital cost = working capital+acquisition rigth+depreciable equipment+30% development.
= $300.000 + $900.000 + $1.000.000 + $360.000 = $2.560.000

e. Cashflow setelah pajak jika amortisasi 30% selama 5 tahun dan deplesi dihitung dengan metode
percentage depletion.

Tahun ke- 0 1 2 3 4 5
Revenue 0 3.000 3.300 3.630 3.993 4.392
-Royalty (15%) 0 -450 -495 -545 -599 -659
Net revenue 0 2.550 2.805 3.086 3.394 3.733
-Operating cost 0 -1.000 -1.080 -1.166 -1.260 -1.360
-Mining tax 0 -30 -30 -30 -30 -30
-Development cost -840 0 0 0 0 0
-Amortization -72 -72 -72 -72 -72 0
-Depreciation 0 -200 -200 -200 -200 -200
-Percentage depletion 0 -382 -421 -463 -509 -560
Taxable income -912 866 1.002 1.155 1.323 1.583
-Tax (30%) 273,6 -259,8 -300,6 -346,5 -396,9 -474,9
Net income -638,4 606,2 701,4 808,5 926,1 1.108,1
+Amortization 72 72 72 72 72 0
+Depreciation 0 200 200 200 200 200
+Depletion 0 382 421 463 509 560
+Working capital return 0 0 0 0 0 300
-Capital cost -2.560 0 0 0 0 0
CASH FLOW -3.126,4 1.260,2 1.394,4 1543,5 1.707,1 2.168,1
Keterangan:
1)
Development cost = 70% x $1.200 = $840
2)
Amortisasi tahun ke-n = 30% (dev. cost)/periode amortisasi
= 30% (1.200.000)/5 = $72.000 (tetap sampai tahun ke-5)
3)
Deprecition tahun ke-n = (P-S)/n
= (1.000.000 0)/5 = $200.000 (tetap selama 5 tahun)
4)
Percentage depletion diperbolehkan jika nilainya lebih kecil dari batas deplesi maksimal:
Batas deplesi maksimal = 50% x taxable income befofe depletion
Batas deplesi th-1 = 50%(2.550.000 - 1.000.000 - 30.000 - 0 72 - 200.000) = $588
Batas deplesi th-2 = 50%(2.805.000 - 1.080.000 - 30.000 0 - 72 - 200.000) = $675,5
Batas deplesi th-3 = 50%(3.086.000 - 1.166.000 - 30.000 0 - 72 - 200.000) = $773
Batas deplesi th-4 = 50%(3.394.000 - 1.260.000 - 30.000 0 - 72 - 200.000) = $880
Batas deplesi th-5 = 50%(3.733.000 - 1.360.000 - 30.000 0 - 72 - 200.000) = $999,5

Percentage depletion = 15% x net revenue


Percentage depletion th-1 = 15% x 2.550.000 = $382.000
Percentage depletion th-2 = 15% x 2.805.000 = $421.000
Percentage depletion th-3 = 15% x 3.086.000 = $463.000
Percentage depletion th-4 = 15% x 3.394.000 = $509.000
Percentage depletion th-5 = 15% x 3.733.000 = $560.000
5)
Tax (30%), jika taxable income negatif maka dianggap sebagai saving.
6)
Capital cost:
Capital cost = working capital+acquisition rigth+depreciable equipment+30% dev.t
= $300.000 + $900.000 + $1.000.000 + $360.000 = $2.560.000

Hasil perhitungan kelima cashflow di atas dapat dirangkum sebagai berikut:

Yuli Kusumawati, Catatan Kuliah Ekonomi Mineral - 47


Cashflow (x$1000) th ke- 0 1 2 3 4 5
Before tax -3.400 1.550 1.725 1.919 2.134 2.673
100% dev.cost, cost depl. -3.040 1.178 1.300,5 1.437 1.586,8 2.054,1
100% dev.cost, 15% depl. -3.040 1.238,6 1.372,8 1.521,9 1.685,5 2.168,1
30% amortization, cost depl. -3.126,4 1.199,6 1.322,1 1.458,6 1.608,4 2.054,1
30% amortization, 15% depl. -3.126,4 1.260,2 1.466,4 1543,5 1.707,1 2.168,1

Untuk mengetahui pengaruh pajak terhadap cashflow dilakukan analisis NPV, ROR, PVR atau BCR, dan PBP
pada discount rate tertentu (misal, i=20%)
a. Cashflow sebelum pajak (x$1000)
NPV = PW revenue - PW cost
NPVA =1.550(P/F,20%,1)+1.725(P/F,20%,2)+1.919(P/F,20%,3)+2.134(P/F,20%,4) +2.673(P/F,20%,5) -
50%
2.303,5 3.400
= 1.550(0,833) + 1.725(0,694) + 1.919(0,579) + 2.134(0,482) + 2.673(0,402) - 3.400
= 2.303,5

ROR adalah nilai0i yang menghasilkan NPV=0, diperoleh dengan cara coba-coba:
Untuk i = 20% diperoleh
20% NPVA = $2.303,5
-257,9 ROR=?
Untuk i = 50%, diperoleh NPVA = -$257,9
Selanjutnya dicari nilai ROR dengan interpolasi:
(ROR-20%)/(2.303,5-0) = (50%-20%)/(2.303,5-(-257,9))
RORA = 20% + (50%-20%)((2.303,5/(2.303,5+257,9))
=46,98%

PVRA = NPVA/PW net CFA (-)


= 2.303,5/-3.400 = 0,68

BCRA = PVRA + 1
= 0,68 + 1 = 1,68

Discounted PBP dengan MARR 20%


Terlebih dulu dihitung aliran kas diskonto dan kumulatifnya:

Tahun CFA P/F,20%,n PW CFA cum PW CFA


0 -3.400,0 1,000 -3.400,0 -3.400,0
1 1.550,0 0,833 1.291,7 -2.108,3
2 1.725,0 0,694 1.197,9 -910,4
3 1.919,0 0,579 1.110,5 200,1
4 2.134,0 0,482 1.029,1 1.229,2
5 2.673,0 0,402 1.074,2 2.303,5

Terlihat bahwa periode pengembalian investasi proyek tersebut antara tahun ke-2 dan ke-3, sehingga PBP
dapat dihitung dengan interpolasi:

Yuli Kusumawati, Catatan Kuliah Ekonomi Mineral - 48


terakhir

+

th berikutnya
CF
Cum .CF
terakhir +
PBP=tahun Cum .CF

910,4
PBP=2+
1.110,5

= 2 + 0,82 = 2,82 tahun.

b. Cashflow setelah pajak jika 100% development cost digunakan untuk pengurang pajak dan deplesi
dihitung dengan metode cost depletion.
NPV = PW revenue - PW cost
NPVB =1.178(P/F,20%,1)+1.300,5(P/F,20%,2)+1.437(P/F,20%,3)+1.586,8(P/F,20%,4)

35% +2.054,1(P/F,20%,5)-3.400
907
= 1.178(0,833) + 1.300,5(0,694) + 1.437(0,579) + 1.586(0,482) + 2.054(0,402) - 3.400
= $907

ROR adalah nilai0i yang menghasilkan NPV=0, diperoleh dengan cara coba-coba:
Untuk i = 20% diperoleh
20% NPVA = $907
-294 ROR=?
Untuk i = 35%, nilai NPVA = -$294
Selanjutnya dicari nilai ROR dengan interpolasi:
(ROR-20%)/(907-0) = (35%-20%)/(907-(-294))
RORB = 20% + (35%-20%)((907/(907+294))
= 31,33%

PVRB = NPVB/PW net CFB (-)


= 907/-3.400 = 0,27

BCRB = PVRB + 1
= 0,27 + 1 = 1,27

Discounted PBP dengan MARR 20%


Terlebih dulu dihitung aliran kas diskonto dan kumulatifnya:

Tahun CFB P/F,20%,n PW CFB cum PW CFB


0 -3.400,0 1,000 -3.400,0 -3.400,0
1 1.178,0 0,833 981,7 -2.418,3
2 1.300,5 0,694 903,1 -1.515,2
3 1.437,0 0,579 831,6 -683,6
4 1.586,8 0,482 765,2 81,6
5 2.054,1 0,402 825,5 907,1

Terlihat bahwa periode pengembalian investasi proyek tersebut antara tahun ke-3 dan ke-4, sehingga PBP
dapat dihitung dengan interpolasi:

Yuli Kusumawati, Catatan Kuliah Ekonomi Mineral - 49


terakhir

+

th berikutnya
CF
Cum .CF
terakhir +
PBP=tahun Cum .CF

683,6
PBP=3+
765,2

= 3 + 0,89 = 3,89 tahun.

c. Cashflow setelah pajak jika 100% development cost digunakan untuk pengurang pajak dan deplesi
dihitung dengan metode percentage depletion.
NPV = PW revenue - PW cost
NPVc

=1.238,6(P/F,20%,1)+1.372,8(P/F,20%,2)+1.521,9(P/F,20%,3)+1.685,5(P/F,20%,4)+2.168,1(P/F,20%,5)-
3.400
35% = 1.238,6 (0,833) + 1.372,8 (0,694) + 1.521,9 (0,579) + 1.685,5 (0,482) + 2.168,1(0,402) - 3.400
1.150,4
= $1.150,4

ROR adalah nilai0i yang menghasilkan NPV=0, diperoleh dengan cara coba-coba:
Untuk i = 20% diperoleh
20% NPVC = $1.150,4
-119,7 ROR=?
Untuk i = 35%, nilai NPVC = -$119,7
Selanjutnya dicari nilai ROR dengan interpolasi:
(ROR-20%)/(1.150,4-0) = (35%-20%)/(1.150,4-(-119,7))
RORC = 20% + (35%-20%)((1.150,4/(1.150,4+119,7))
= 33,59%

PVRC = NPVC/PW net CFC (-)


= 1.150,4/-3.400 = 0,34

BCRC = PVRC + 1 = 0,34 + 1 = 1,34

Discounted PBP dengan MARR 20%


Terlebih dulu dihitung aliran kas diskonto dan kumulatifnya:

Tahun CFC P/F,20%,n PW CFC cum PW CFC


0 -3.400,0 1,000 -3.400,0 -3.400,0
1 1.238,6 0,833 1.032,2 -2.367,8
2 1.372,8 0,694 953,3 -1.414,5
3 1.521,9 0,579 880,7 -533,8
4 1.685,5 0,482 812,8 279,1
5 2.168,1 0,402 871,3 1.150,4

Yuli Kusumawati, Catatan Kuliah Ekonomi Mineral - 50


Terlihat bahwa periode pengembalian investasi proyek tersebut antara tahun ke-3 dan ke-4, sehingga PBP
dapat dihitung dengan interpolasi:

terakhir

+

th berikutnya
CF
Cum .CF
terakhir +
PBP=tahun Cum .CF

533,8
PBP=3+
812,8

= 3 + 0,66 = 3,66 tahun.

d. Cashlow setelah pajak jika tingkat amortisasi 30% selama 5 tahun dan deplesi dihitung dengan metode
cost depletion.
NPV = PW revenue - PW cost
NPVD

=1.199,6(P/F,20%,1)+1.322,1(P/F,20%,2)+1.458,6(P/F,20%,3)+1.608,4(P/F,20%,4)+2.054,1(P/F,20%,5) -
3.126,4
40%
1.236,6 = 1.199,6(0,833) + 1.322,1(0,694) + 1.458,6(0,579) + 1.608,4(0,482) + 2.054,1(0,402) - 3.126,4
= $1.236,6

ROR adalah nilai0i yang menghasilkan NPV=0, diperoleh dengan cara coba-coba:
Untuk i = 20% diperoleh
20% NPVD = $1.236,6
-78,6 ROR=?
Untuk i = 40%, nilai NPVD = -$78,6
Selanjutnya dicari nilai ROR dengan interpolasi:
(ROR-20%)/(1.236,6-0) = (40%-20%)/(1.236,6-(-78,6))
RORD = 20% + (40%-20%)((1.236,6/(1.236,6+78,6))
= 38,8%

PVRD = NPVD/PW net CFD (-)


= 1.236,6/-3.126,4 = 0,4

BCRD = PVRD + 1
= 0,4 + 1 = 1,4

Discounted PBP dengan MARR 20%


Terlebih dulu dihitung aliran kas diskonto dan kumulatifnya:

Tahun CFD P/F,20%,n PW CFD cum PW CFD


0 -3.126,4 1,000 -3.126,4 -3.126,4
1 1.199,6 0,833 999,7 -2.126,7
2 1.322,1 0,694 918,1 -1.208,6
3 1.458,6 0,579 844,1 -364,5

Yuli Kusumawati, Catatan Kuliah Ekonomi Mineral - 51


4 1.608,4 0,482 775,7 411,1
5 2.054,1 0,402 825,5 1.236,6

Terlihat bahwa periode pengembalian investasi proyek tersebut antara tahun ke-3 dan ke-4, sehingga PBP
dapat dihitung dengan interpolasi:

terakhir

+

th berikutnya
CF
Cum .CF
terakhir +
PBP=tahun Cum .CF

364,5
PBP=3+
775,7

= 3 + 0,47 = 3,47 tahun.

e. Cashlow setelah pajak jika tingkat amortisasi 30% selama 5 tahun dan deplesi dihitung dengan metode
percentage depletion.
NPV = PW revenue - PW cost
NPVE =
1.260,2(P/F,20%,1)+1.466,4(P/F,20%,2)+1.543,5(P/F,20%,3)+1.707,1(P/F,20%,4)+2.168,1(P/F,20%,5) -
3.126,4
45%
1.529,9 = 1.260,2(0,833) + 1. 466,4(0,694) + 1. 543,5(0,579) + 1. 707,1(0,482) + 2. 168,1(0,402) - 3.126,4
= $1.529,9

ROR adalah nilai0i yang menghasilkan NPV=0, diperoleh dengan cara coba-coba:
Untuk i = 20% diperoleh
20% NPVE = $1.529,9
-329,1 ROR=?
Untuk i = 40%, nilai NPVE = -$329,1
Selanjutnya dicari nilai ROR dengan interpolasi:
(ROR-20%)/(1.529,9-0) = (45%-20%)/(1.529,9-(-329,1))
RORE = 20% + (45%-20%)((1.529,9/(1.529,9+329,1))
= 40,6%

PVRE = NPVE/PW net CFE (-)


= 1.529,9/-3.126,4 = 0,5

BCRE = PVRE + 1
= 0,5 + 1 = 1,5

Discounted PBP dengan MARR 20%


Terlebih dulu dihitung aliran kas diskonto dan kumulatifnya:

Tahun CFE P/F,20%,n PW CFE cum PW CFE


0 -3.126,4 1,000 -3.126,4 -3.126,4
1 1.260,2 0,833 1.050,2 -2.076,2

Yuli Kusumawati, Catatan Kuliah Ekonomi Mineral - 52


2 1.466,4 0,694 1.018,3 -1.057,9
3 1.543,5 0,579 893,2 -164,7
4 1.707,1 0,482 823,3 658,6
5 2.168,1 0,402 871,3 1.529,9

Terlihat bahwa periode pengembalian investasi proyek tersebut antara tahun ke-3 dan ke-4, sehingga PBP
dapat dihitung dengan interpolasi:

terakhir

+

th berikutnya
CF
Cum .CF
terakhir +
PBP=tahun Cum .CF

164,7
PBP=3+
823,3

= 3 + 0,20 = 3,20 tahun.

Hasil analisis kelima cashflow di atas dapat dirangkum sebagai berikut:

NPV IRR PBP


Cashflow PVR BCR
(X$1.000) (%) (tahun)
Before tax 2.303,5 46,98 0,68 1,68 2,82
100% dev.cost, cost depl. 907,1 31,33 0,27 1,27 3,89
100% dev.cost, 15% depl. 1.150,4 33,59 0,34 1,34 3,66
30% amortization, cost depl. 1.236,6 38,80 0,40 1,40 3,47
30% amortization, 15% depl. 1.529,9 40,57 0,49 1,49 3,20

Kesimpulan:
Investasi yang tidak dikenakan pajak memberikan keuntungan terbesar secara ekonomi.Semakin besar
pengurangan pajak di awal tahun, semakin besar pula keuntungan yang bisa diperoleh kaitannya dengan
ekonomi proyek.

Yuli Kusumawati, Catatan Kuliah Ekonomi Mineral - 53


9. ANALISIS PENGGANTIAN
Tujuan:
Memahami konsep penggantian (replacement) aset.
Mampu menghitung umur ekonomi aset.
Mampu menganalisis sewa (leasing) dan beli (purchasing) suatu aset.

9.1. Konsep penggantian


Penggantian (replacement) suatu aset dilakukan jika aset tersebut sudah tidak menguntungkan secara
ekonomi jika dibandingkan dengan menggunakan aset baru dengan fungsi sama tetapi memberikan layanan
yang lebih baik.
Alasan penggantian aset antara lain:
Untuk meningkatkan kapasitas produksi.
Aset lama sudah rusak atau memerlukan biaya perbaikan yang besar.
Aset lama sudah usang sehingga tidak bisa bekerja dengan efisien.
Konsep dasar analisis penggantian adalah membandingkan total nilai tahunan aset petahana (defender)
dengan total nilai tahunan aset pengganti (challenger).

9.2. Umur ekonomis


Pada umumnya aset mempunyai komponen biaya sebagai berikut:
Capital recovery cost, dihitung dari harga beli alat.
Biaya rata-rata operasi dan perawatan
Biaya total yang merupakan jumlah dari capital recovery cost dengan rata-rata biaya perawatan.

Gambar 9.1 Hubungan biaya dengan umur aset.

Pada gambar di atas terlihat captal recovery cost


mengalami penurunan seiring bertambahnya umur
mesin, sementara biaya operasi dan perawatan
mengalami kenaikan seiring bertambahnya umur
mesin. Pada awalnya total biaya menurun sampai pada
umur mesin tertentu kemudian bergerak naik. Titik
dimana total biaya adalah minimum disebut umur economi dari suatu aset.

Contoh 9-1:
Sebuah perusahaan bermaksud mengganti peralatan dengan biaya awal $4.000 dengan nilai sisa nol.
Berdasarkan pengalaman biaya perawatan selama tahun ke-1 adalah nol dan setelah itu akan naik $200/th.
Kapan peralatan tersebut akan diganti jika i = 12%?
Jawab:
Hasil hitungan disajikan dalam tabel berikut ini:

Yuli Kusumawati, Catatan Kuliah Ekonomi Mineral - 54


Biaya awal (harga peralatan) = $4.000.
(A) tahun pemakaian alat
(B) biaya perawatan tahun ke-1 = 0, tahun berikutnya naik $200/th.
(C) nilai faktor (P/F,12%,n).
(D) nilai sekarang dari biaya perawatan: PWmc = mc(P/F,i,n).
(E) jumlahkan biaya perawatanselama tahun yang diberikan.
(F) tambahkan kumulatif biaya perawatan dengan biaya awal.
(G) nilai faktor (A/P,12%,n).
(H) kalikan hasil hitungan kolom (F) dengan kolom (G). Selanjutnya dari hasil biaya total tahunan minimum
dilihat berada pada akhir tahun ke berapa, yang merupakan titik dimana suatu aset mencapai umur
ekonominya. Pada contoh ini nilai ekonomi peralatan adalah 7 tahun.

9.3. Membandingkan sewa atau beli


Analisis sewa (leasing) atau beli (purchasing) suatu aset berkaitan dengan:
Penentuan biaya penggunaan suatu aset yang paling minimum.
Perhitungan cashflow sesudah pajak.

Sewa (leasing) adalah suatu alternatif penggunaan aset selama periode tertentu tanpa harus memilikinya.
Akan tetapi secara analisis sewa bisa berbentuk sebagai berikut:
a. Operating lease (sewa operasi), adalah perjanjian sewa atas penggunaan aset oleh penyewa
(lessee/user) selama periode tertentu yang disepakati (aplikasi sehari-hari seperti menyewa mobil di
rental).
Ketentuan dalam operating lease adalah sebagai berikut:
Pihak pemilik (lessor) menguasai kepemilikan aset secara penuh.
Pihak pemilik menanggung biaya perawatan dan perbaikan aset.
Pihak penyewa bisa membatalkan perjanjian karena alasan tertentu,tetapi harus membayar pinalti.
Pembayaran operating lease dapat digunakan seluruhnya untuk pengurang penghasilan pajak pada
saat pembayaran tersebut dilakukan.
Karena penyewa tidak punya kepemilikan apapun atas aset yang disewa, maka tidak ada capital
cost, depresiasi, dan nilai sisa aset di pihak penyewa.
b. Capital lease/financial lease (sewa modal), adalah alternatif untuk memiliki aset dengan cara membayar
angsuran pinjaman. Dengan kata lain pengguna aset (lessee) mendapat pinjaman modal dari dealer
(lessor) yang menjual aset tersebut, dan karenanya pengguna aset harus membayar angsuran untuk
bisa memiliki aset tersebut.
Ketentuan dalam capital lease adalah sebagai berikut:

Yuli Kusumawati, Catatan Kuliah Ekonomi Mineral - 55


Pembayaran sewa tidak bisa seluruhnya digunakan sebagai pengurang pajak pada saat
pembayaran dilakukan.
Komponen bunga yang berhubungan dengan pembayaran sewa bisa dijadikan pengurang pajak.
Nilai sekarang dari pembayaran capital lease bisa didepresiasi selama umur perolehan.
Terdapat nilai sisa pada akhir periode layanan, karena penyewa akan memiliki aset tersebut di akhir
periode capital lease.
c. Leverage lease, yaitu alternatif pemilikan aset yang dibiayai pihak ketiga (bukan dealer penjual aset).
Ketentuan leverage lease bisa merujuk pada operating lease atau capital lease.

Contoh 9-2:
Analisislah alternatif pengadaan aset berikut ini untuk waktu dua tahun dengan asumsi MARR = 15%,
income tax rate = 40%, depresiasi straigth line dengan umur aset 2 tahun, dan kondisi expense against
other income.
A) Pembelian tunai
C=350 - L=150

0 1 2
B) Operating lease
LP=60 LP=120 LP=60

0 1 2
C) Capital lease L=150
LP=100 LP=200 LP=100

0 1 2
D) Pembelian dengan 80% dana berasal dari pinjaman, bunga pinjaman = 9%/th
C=350 L=150
B=280 MP=159 MP=159

0 1 2

Keterangan:
Semua nilai dikali $1.000, C = capital cost (biaya modal), L = salvage value (nilai sisa), LP = lease payment
(pembayaran sewa), B = borrowed money (uang yang dipinjam), MP = mortgage payment (pembayaran
angsuran).
Jawab:
Semua hitungan dalam ribuan dollar.
A) Analisis cashflow pembelian tunai

Tahun ke- 0 1 2
Salvage 0 0 150
-Depreciation* 0 -100 -100
Taxable income 0 -100 -100
-Tax 40% ** 0 40 40
Net income 0 -60 -60
+Depreciation 0 100 100
-Capital cost -350 0 0
Cashflow -350 40 40

PW cost A = -350 + 40(P/F,15%,1) + 40(P/F,15%,2)


= -350 + 40(0,8696) + 40(0,7561)= -$217
Keterangan:
Karena aset tersebut sepenuhnya milik pembeli, maka ada capital cost, ada depresiasi, dan ada salvage
value.
*Depresiasi straigth line:
D = (C-L)/n = (350-150)/2 = 100 (sama untuk tahun ke-1 dan ke-2)
**Jika taxable income hasilnya negatif maka pajak merupakan saving (positif).

Yuli Kusumawati, Catatan Kuliah Ekonomi Mineral - 56


B) Analisis cashflow operating lease

Tahun ke- 0 1 2
Salvage 0 0 0
-Lease payments -60 -120 -60
Taxable income -60 -120 -60
-Tax 40% * 24 48 24
Net income -36 -72 -36
-Capital cost 0 0 0
Cashflow -36 -72 -36

PW cost B = -36-72(P/F,15%,1) -36(P/F,15%,2)


= -36 -72(0,8696) -36(0,7561)= -$126
Keterangan:
Karena penyewa tidak punya kepemilikan atas aset tersebut, maka tidak ada capital cost, tidak ada
depresiasi, dan tidak ada salvage value.
*Jika taxable income hasilnya negatif maka pajak merupakan saving (positif).

C) Analisis cashflow capital lease

Tahun ke- 0 1 2
Salvage 0 0 150
-Interest1) 0 -24 -8
-Depreciation2) 0 -109 -109
Taxable income 0 -133 33
-Tax 40%3) 0 53 -13
Net income 0 -80 20
+Depreciation 0 109 109
-Capital cost4) -100 -176 -92
Cashflow -100 -147 37

PW cost C = -100 -147(P/F,15%,1) +37(P/F,15%,2)


= -100 -147(0,8696) +37(0,7561)= -$200
Keterangan:
Karena aset tersebut menjadi milik pembeli setelah pembayaran angsuran lunas, maka ada capital cost, ada
depresiasi, dan ada salvage value.
Seluruh pembayaran angsuran dibawa ke nilai sekarang (PW of capital lease payment) dengan diskon sesuai
bunga pinjaman (dalam kasus ini i = 9%)
PW cap. lease payment = 100 + 200(P/F,9%,1) + 100(P/F,9%,2)
= 100 + 200(0,9174) + 100(0,8417)= $368

1)Interest dihitung dengan cara sebagai berikut:


Principal Payment Interest Principal reducing Balance
A B C = 9% x A D=BC E=AD
Th-0 368 100 0 100 268
Th-1 268 200 24 176 92
Th-2 92 100 8 92 0
Pada th-0 seluruh pembayaran angsuran (payment) digunakan untuk mengurangi pinjaman (principal),
sehingga interest sama dengan nol.
2) Basis untuk perhitungan depresiasi adalah PW cost of capital lease payment (P).

Yuli Kusumawati, Catatan Kuliah Ekonomi Mineral - 57


Depresiasi straigth line:
D = (P-L)/n = (368-150)/2 = 109 (sama untuk tahun ke-1 dan ke-2)
3)Jika taxable income hasilnya negatif maka pajak merupakan saving (positif).
4)Capital cost = payment - interest

D) Analisis cashflow pembelian dengan pinjaman 80%.

Tahun ke- 0 1 2
Salvage 0 0 150
-Interest1) 0 -25 -13
-Depreciation2) 0 -100 -100
Taxable income 0 -125 37
-Tax 40% 3) 0 50 -15
Net income 0 -75 22
+Depreciation 0 100 100
+Loan amount 80% 280 0 0
-Principal payment4) 0 -134 -146
-Capital cost -350 0 0
Cashflow -70 -109 -24

PW cost A = -70 -109(P/F,15%,1) -24(P/F,15%,2)


= -70 -109(0,8696) -24(0,7561) = -$183
Keterangan:
Karena aset tersebut menjadi milik pembeli setelah pembayaran angsuran lunas, maka ada capital cost, ada
depresiasi, dan ada salvage value.
Seluruh pembayaran angsuran dibawa ke nilai sekarang (PW of capital lease payment) dengan diskon sesuai
bunga pinjaman (dalam kasus ini i = 9%)
PW cap. lease payment = 100 + 200(P/F,9%,1) + 100(P/F,9%,2)
= 100 + 200(0,9174) + 100(0,8417) = $368
1)Interest dihitung dengan cara sebagai berikut:
Principal Payment Interest Principal reducing Balance
A B C = 9% x A D=BC E=AD
Th-0 280 0 0 0 280
Th-1 280 159 25 134 146
Th-2 146 159 13 146 0
Pada th-0 seluruh pembayaran angsuran (payment) digunakan untuk mengurangi pinjaman (principal),
sehingga interest sama dengan nol.
2)Basis untuk perhitungan depresiasi adalah capital cost.
Depresiasi straigth line:
D = (C-L)/n = (350-150)/2 = 100 (sama untuk tahun ke-1 dan ke-2)
3)Jika taxable income hasilnya negatif maka pajak merupakan saving (positif).
4)Principal payment = payment - interest

Yuli Kusumawati, Catatan Kuliah Ekonomi Mineral - 58


10. ANALISIS RESIKO DAN KETIDAKPASTIAN
Tujuan:
Memahami pengaruh resiko dan ketidakpastian dalam investasi.
Mampu menganalisis sensitivitas perubahan parameter-parameter yang berpengaruh terhadap
keuntungan suatu investasi.
Mampu melakukan pengambilan keputusan dengan pohon keputusan.

Pada umumnya investasi dalam proyek berlaku untuk jangka waktu yang panjang. Selama jangka waktu
tersebut mungkin terdapat perubahan dari iklim usaha yang sulit diramalkan, karena banyaknya faktor-
faktor yang mempengaruhi baik faktor internal maupun faktor eksternal dari proyek atau perusahaan.
Adanya unsur-unsur ketidakpastian dalam jangka waktu tertentu harus diperhitungkan dalam analisis
ekonomi agar diperoleh hasil yang lebih akurat.

Manajemen resiko antara lainmeliputi:


Analisis sensitivitas dari parameter-parameter yang mempengaruhi keuntungan.
Pengambilan keputusan dengan pohon keputusan (decision trees)
Simulasi dengan menggunakan bilangan acak (random numbers)
Presentasi hasil analisis di atas, untuk memudahkan pihak manajemen dalam pengambilan keputusan.

10.1. Analisis sensitivitas


Analisis sensitivitas adalah evaluasi terhadap pengaruh perubahan parameter-parameter yang
mempengaruhi keuntungan.Parameter-parameter yang dianalisis sensitivitasnya antara lain:
Investasi awal (initial investment)
Harga jual (selling price)
Biaya operasi (operating cost)
Umur proyek (project life)
Nilai sisa (salvage value)
Tingkat pengembalian minimum (MARR)

Kelebihan analisis sensitivitas antara lain:


Membantu mengidentifikasi parameter-parameter yang sangat mempengaruhi keuntungan berdasarkan
tingkat perubahan keuntungan yang diakibatkannya.
Mudah dilakukan dengan komputer.

Kelemahan analisis sensitivitas antara lain:


Tidak memberikan indikasi kemungkinan (likehood) sesuatu yang diandaikan terjadi. Misal, berapa
kemungkinan harga turun 20%?
Tidak memperlihatkan ketergantungan antar parameter-parameter yang mempengaruhi keuntungan.

10.1.1. Analisis sensitivitas variabel tunggal


Contoh 10-1:
Suatu proyek memerlukan investasi awal pada tahun ke-0 sebesar $150.000. Proyek tersebut akan
menghasilkan manfaat $100.000 /tahun dengan biaya operasi $50.000 mulai tahun ke-1 sampai tahun ke-5.

Yuli Kusumawati, Catatan Kuliah Ekonomi Mineral - 59


Nilai sisa dari aset yang didepresiasi $70.000 pada akhir tahun ke-5. Tingkat pengembalian yang diharapkan
adalah 20%. Evaluasi sensitivitas NPV proyek dengan rentang -20%, -40%, +20% dan +40% terhadap:
a. Investasi awal (C)
b. Nilai manfaat tahunan (B)
c. Biaya operasi tahunan (OC)
d. Nilai sisa (S)
e. Umur proyek (n)
f. Tingkat pengembalian minimum (MARR)

Jawab:
Cashflow dari proyek tersebut adalah sebagai berikut:

Tahun ke- 0 1 2 3 4 5
Investasi -$150.000 0 0 0 0 0
Manfaat 0 $100.000 $100.000 $100.000 $100.000 $100.000
Biaya operasi 0 -$50.000 -$50.000 -$50.000 -$50.000 -$50.000
Nilai sisa 0 0 0 0 0 $70.000

Untuk MARR = 20%, dan umur investasi 5 tahun:


NPV = -Investasi awal + Manfaat/th (P/A,20%,5) Biaya/th (P/A,20%,5) + Nilai sisa (P/F,20%,5)
NPV = -150.000 + 100.000(P/A,20%,5) -50.000(P/A,20%,5)+ 70.000(P/F,20%,5)
NPV = -150.000 + 100.000(2,9906) - 50.000(2,9906) + 70.000(0,4019) = $27.663

a. Analisis sensitivitas terhadap investasi awal (C).

Investasi awal ($) Perubahan prediksi (%) NPV ($) Perubahan thd NPV (%)
90.000 -40 87.663 217
120.000 -20 57.663 108
150.000 0 27.663 0
180.000 +20 -2.337 -108
210.000 +40 -32.337 -217

b. Analisis sensitivitas terhadap manfaat tahunan (B).

Manfaat/th($) Perubahan prediksi (%) NPV ($) Perubahan thd NPV (%)
60.000 -40 -91.961 -432
80.000 -20 -32.149 -216
100.000 0 27.663 0
120.000 +20 87.475 216
140.000 +40 147.287 432

c. Analisis sensitivitas terhadap biaya operasi tahunan (OC).

Biaya operasi/th($) Perubahan prediksi (%) NPV ($) Perubahan thd NPV (%)
30.000 -40 87.475 216
40.000 -20 57.569 108
50.000 0 27.663 0
60.000 +20 -2.243 -108
70.000 +40 -32.149 -216

d. Analisis sensitivitas tehadap nilai sisa (S).

Nilai sisa($) Perubahan prediksi (%) NPV ($) Perubahan thd NPV (%)
42.000 -40 16.410 -41

Yuli Kusumawati, Catatan Kuliah Ekonomi Mineral - 60


56.000 -20 22.036 -20
70.000 0 27.663 0
84.000 +20 33.290 20
98.000 +40 38.916 41

e. Analisis sensitivitas tehadap umur proyek.

Umur proyek (n) Perubahan prediksi (%) NPV ($) Perubahan thd NPV (%)
3 -40 -4.166 -115
4 -20 13.196 -52
5 0 27.663 0
6 +20 39.718 44
7 +40 49.767 80

f. Analisis sensitivitas tehadap tingkat pengembalian minimum (MARR).

MARR (%) Perubahan prediksi (%) NPV ($) Perubahan thd NPV (%)
12 -40 69.958 153
16 -20 47.042 70
20 0 27.663 0
24 +20 11.147 -60
28 +40 -3.030 -111

Dari hasil analisis di atas terlihat bahwa pengaruh terhadap NPV akibat perubahan dari investasi awal,
manfaat tahunan, dan biaya operasi tahunan lebih signifikan daripada akibat perubahan nilai sisa. Dengan
kata lain pengaruh perubahan parameter-parameter yang dekat dengan tahun ke-0 lebih signifikan daripada
pengaruh akibat perubahan parameter yang terletak beberapa tahun berikutnya dari tahun ke-0.
Sedangkan perubahan umur proyek dan tingkat pengembalian minimum benar-benar menyebabkan
perubahan dalam keseluruhan cashflow.

Hasil analisis dapat digambarkan dalam grafik Strauss berikut ini:

500
400
Investasi aw al
300
Manfaat/th
200
Linear (Manfaat/th)
100 Biaya operasi/th
0 Linear (Biaya operasi/th)
-100-40% -20% 0 +20% +40% Nilai sisa
Umur proyek
-200
MARR
-300
Linear (MARR)
-400
-500

10.1.2. Analisis sensitivitas dengan pendekatan rentang

Yuli Kusumawati, Catatan Kuliah Ekonomi Mineral - 61


Pendekatan rentang berkaitan dengan perkiraan nilai yang paling optimis (yang terbaik) dan nilai yang
paling pesimis (yang terburuk) untuk masing-masing faktor di samping perkiraan nilai yang paling
diharapkan.

Pendekatan ini memudahkan pembuatan keputusan investasi untuk kasus dimana:


a. Suatu proyek terlihat menarik, meskipun saat nilai pesimis digunakan, sehingga harus diambil.
b. Suatu proyek terlihat tidak menarik, meskipun saat nilai optimis digunakan, sehingga harus ditolak.

Ketika suatu proyek terlihat baik dengan nilai optimis tetapi terlihat buruk dengan nilai pesimis, maka kajian
lebih lanjut terhadap ketidakpastian dan resiko proyek tersebut harus dilakukan.

Contoh 10-2:
Cashflow dari suatu proyek adalah sebagai berikut:

Tahun ke- 0 1 2 3 4 5
Investasi $150.000 0 0 0 0 0
Manfaat 0 $100.000 $100.000 $100.000 $100.000 $100.000
Biaya operasi 0 $50.000 $50.000 $50.000 $50.000 $50.000
Nilai sisa 0 0 0 0 0 $70.000
Cashflow $150.000 $50.000 $50.000 $50.000 $50.000 $120.000

Jika rentang perubahan -20% dan +20% terjadi pada investasi awal, manfaat tahunan, biaya operasi, dan
nilai sisa, lakukan analisis sensitivitas untuk kondisi terbaik dan kondisi teburuk dengan umur proyek
masing-masing 5 tahun dan MARR 20%.

Jawab:

Kondisi terbaik kondisi yang diharapkan Kondisi terburuk


Investasi $120.000 $150.000 $180.000
Manfaat $120.000 $100.000 $80.000
Biaya operasi $40.000 $50.000 $60.000
Nilai sisa $84.000 $70.000 $56.000
Umur proyek 5 th 5 th 5 th
NPV $153.048 $27.690 -$97.668

Hasil analisis memperlihatkan bahwa proyek memuaskan (layak) pada kondisi terbaik dan pada kondisi yang
diharapkan, tetapi tidak memuaskan pada kondisi terburuk.

10.2. Analisis probabilitas


10.2.1. Teori probabilitas

Yuli Kusumawati, Catatan Kuliah Ekonomi Mineral - 62


Teori probabilitas digunakan untuk meramalkan kejadian-kejadian yang akan timbul di masa datang dengan
resiko (derajat kepercayaan) tertentu. Keputusan yang diambil berdasarkan teori probabilitas digolongkan
sebagai proses pengambilan keputusan berdasarkan resiko tertentu dan diharapkan dapat mendekati situasi
yang sebenarnya di masa datang. Komponen teori probabilitas yang banyak digunakan dalam pengambilan
keputusan adalah nilai ekspektasi (rata hitung) dan varian.
Untuk suatu variabel random x:
a. Nilai ekspektasi :
Dimana: P(x) = fungsi probabilitas masa dari x
E(x) = fungsi pada probabilitas dari x
b. Nilai varian:

Contoh 10-3:
Andaikan dua mata uang yang seimbang dilempar, berapa nilai ekspektasi dan varian dari jumlah tampak
muka yang akan terjadi?
Jawab:
Misalnya:
M1 = uang logam pertama menunjukkan tampak muka
B1 = uang logam pertama menunjukkan tampak belakang
M2 = uang logam kedua menunjukkan tampak muka
B2 = uang logam kedua menunjukkan belakang

Ruang eksperimen dari pelemparan dua uang logam tersebut adalah:


S = {(M1 M2), (M1 B2), (B1 M2), (B1 B2)}

Jika x merupakan variabel acak yang menunjukkan banyaknya tampak muka yang muncul, akan diperoleh:

P(x=0) = P(B1 B2) = P (B1) P(B2) = (0,5) (0,5) = 0,25


P(x=1) = P(M1 B2) = P(B1 B2 B1 M2) = P(M1) P(B2) + P(B1) P(M2)
= (0,5) (0,5) + (0,5) (0,5) = 0,50
P(x=2) = P(B1 B2) = P(b1) P(B2) = (0,5) (0,5) = 0,25

Tabel distribusi probabilitasnya:

Contoh 10-4:
Dari contoh 10-3 andaikan diadakan aturan permainan sebagai berikut:

Yuli Kusumawati, Catatan Kuliah Ekonomi Mineral - 63


Jika tidak ada yang muncul maka kalah Rp 100,00
Jika tampak muka 1 yang muncul maka menang Rp 40,00
Jika tampak muka 2 yang muncul maka menang Rp 80,00
Maka nilai kemenangan atau keuntungan yang diharapkan (nilai ekspektasi) dari permainan tersebut jika
permainan berlangsung lama adalah:
E(keuntungan) = - Rp 100 (0,25) + Rp 40 (0,50) + Rp 80 (0,25) = Rp 15,00

10.2.2. Pohon keputusan


Pohon keputusan adalah skema rangkaian keadaan dan kemungkinan (probabilitas) hasilnya.
Pohon keputusan digunakan dalam pengambilan keputusan dengan kondisi antara lain:
Terdapat banyak alternatif yang harus dipilih
Terdapat banyak kemungkinan kondisi/keadaan yang dapat terjadi (baik, sedang, buruk)
Pengambilan keputusan memerlukan proses bertahap/bersyarat (bayesian approach).

Aturan-aturan dalam pohon keputusan adalah:


Pohon keputusan dibaca dari kiri ke kanan.
Ujung cabang disebut terminal.
Nilai kondisi (condition value) diberikan di setiap titik terminal. Nilai ini biasanya merupakan nilai
sekarang bersih (net present value).
Titik keputusan (decision node) disimbolkan dengan kotak.
Titik kemungkinan disimbolkan dengan lingkaran.
Kemungkinan terdapat dua atau lebih titik kemungkinan yang berurutan.
Probabilitas harus diberikan di setiap cabang dari titik kemungkinan (chance node) dimana jumlah
probabilitas di sekitar titik tersebut adalah satu.
Tidak ada skala untuk menggambar pohon keputusan.

Cara menyelesaikan pohon keputusan:


Mulai dari titik terminal dan bekerja mundur ke awal keputusan. Jika titik kemungkinan dicapai, hitung
nilai harapan (expected monetary value/EMV) untuk semua nilai kondisional dan tulis di atas titik
kemungkinan.
EMV = p,f dan p =1
Keterangan:
EMV = nilai harapan
p = probabilitas
f = nilai kondisional
i = nomor cabang
n = jumlah cabang
Jika sampai pada titik keputusan EMV yang terbesar, coret pilihan lain dan letakkan EMV di atas ttitik
keputusan. Selanjutnya mundur sampai titik keputusan awal dicapai. Pilih jalur dengan EMV terbesar
untuk memilih keuntungan dan EMV terkecil untuk memilih biaya.

Contoh 10-5:
Sebuah perusahaan dihadapkan pada dua keputusan apakah akan melakukan pemboran atau tidak terhadap
lahan pertambangan minyak yang dimilikinya. Jika dilakukan pemboran akan menelan biaya $1 juta dengan
kemungkinan gagal adalah 0,8 dan kemungkinan berhasil adalah 0,2,sedangkan perolehan discounted NPV
adalah $5 juta. Tentukan keputusan yang akan diambil dengan menggunakan pohon keputusan.
Jawab:
EMV membor = (0,8 x -$1 juta) + (0,2 x $5 juta) = Kering
$0,2 juta NPV = -$1 juta
Probabilitas = 0,8
EMV tidak megebor = 0
Keputusan: mengebor membor

Produksi
NPV = $5 juta
Probabilitas = 0,2
tidak membor
NPV = 0
Yuli Kusumawati, Catatan Kuliah Ekonomi Mineral - 64
REFERENSI
1. Grant, E.L., Ireson, W.G., and Leavenworth, R.S., 1982, Principles of Engineering Economy 7th edition,
John Wiley and Sons, Singapore.
2. Mangitung, D.M., 2013, Ekonomi Rekayasa, Disertai Cara Penyelesaian Dengan Speadsheet, Penerbit
Andi, Yogyakarta.
3. Panneerselvam, R., 2012, Engineering Economics, PHI Learning Private Limited, New Delhi.
4. Partowidagdo, W., 2001, Risk Analysis & Project Risk Management, Institut Teknologi Bandung.
7. Stermole, F.J., Stermole J.M., 1996, Economic Evaluation and Investment Decision Methods 9th
edition, Investment Evaluations Corporation, Colorado.

Yuli Kusumawati, Catatan Kuliah Ekonomi Mineral - 65

Anda mungkin juga menyukai